Download A 63-year-old man comes to the clinic complaining of acute left toe

Document related concepts

Signs and symptoms of Graves' disease wikipedia , lookup

Hyperthyroidism wikipedia , lookup

Graves' disease wikipedia , lookup

Diabetic ketoacidosis wikipedia , lookup

Diabetes in dogs wikipedia , lookup

Complications of diabetes mellitus wikipedia , lookup

Transcript
A 63-year-old man comes to the clinic complaining of acute left toe pain that
began last night while he was lying in bed. He rates the pain at 9 out of 10, on
a pain scale, with 10 being the worst pain, and he tells you that it is much
worse when his bed sheets rub across the toe. His past medical history is
significant for gastroesophageal reflux disease for which he takes omeprazole,
and he was recently diagnosed with mild hypertension for which he was started
on hydrochlorothiazide. His blood pressure is 125/85 mm Hg, pulse is 82/min,
and respirations are 18/min. He has swelling and erythema over the first
metatarsophalangeal (MTP) joint of the left foot. The remainder of his
examination, including other joints is entirely normal. Laboratory studies show
a normal complete blood count, uric acid 12.4 mg/dl, and creatinine 0.9 mg/dl.
An arthrocentesis is performed which shows negatively birefringent crystals.
The most appropriate management for this patient is
A. allopurinol 100 mg orally once daily then a follow up uric acid level in 1
month
B. ibuprofen 600 mg orally every 8 hours for 4 weeks and start allopurinol
100 mg once daily as soon as symptoms have resolved. Follow up uric acid
level in 1 month
C. ibuprofen 600 mg orally every 8 hours until symptoms resolve and
change his antihypertensive agent. Follow up uric acid level in 1 month
D. no treatment is necessary for this self-limited condition
E. probenecid 250 mg orally once daily then follow up uric acid level in 1
month
Explanation:
The correct answer is C. This patient is presenting with podagra, the term
applied to acute gouty arthritis of the first MTP. This is a common disease
associated with elevated uric acid levels in the blood, and typically seen in
elderly individuals. One of the most common predisposing factors is the recent
institution of a thiazide diuretic. This impairs the renal excretion of uric acid,
elevating their blood levels. Many other drugs are associated with
hyperuricemia and it can be idiopathic. He should be treated with ibuprofen
and his antihypertensive agent should be changed.
The main tenets of treatment are to look for underlying causes, and then
symptomatically treat the acute attack. If no predisposing, and alterable factors
can be found, it is often helpful to start the patient on allopurinol to prevent
future attacks. It is important, however, not to start allopurinol early in an acute
attack, particularly, without an anti-inflammatory drug in combination (choice
A). This can actually exacerbate the patient's symptoms.
Ibuprofen 600 mg orally every 8 hours for 4 weeks and starting allopurinol 100
mg once daily as soon as symptoms have resolved, with follow up of uric acid
level within 1 month (choice B) is not the most appropriate management for
this patient. Even though this is a very typical scenario in the treatment of an
acute gouty attack, given the fact that the patient was recently started on
hydrocholorothiazide, it is reasonable to try changing this medicine first, rather
than adding another medication (allopurinol) to his regimen.
It is true that an acute gouty attack is often self-limited, (choice D). However,
it is not appropriate to let the patient suffer pain any longer than he has to.
Non-steroidal anti-inflammatory drugs such as ibuprofen will not only help
with symptoms, but they will also shorten the duration of the attack.
Probenecid (choice E) is another drug used in the long-term treatment of gout.
It actually increases the excretion of uric acid from the kidneys. 24-hour uric
acid excretion studies can be performed to identify patients who are so called
"underexcretors". They may be better served by treatment with this medication
rather than allopurinol. In either case, it is not the best answer in this question
for the reasons described above.
A 33-year-old woman comes to the office because of generalized weakness
and a "pins and needles" feeling in her lower extremities for the past 3 weeks.
She states that she feels "unsteady" on her feet. She exercises daily, rarely
drinks alcohol, and is a strict vegetarian. Since this is the first time you have
met this woman, she tells you that she has not had any major illnesses, but has
been hospitalized multiple times over the past few years for anorexia nervosa.
Her temperature is 37 C (98.6 F), blood pressure is 110/70 mm Hg, pulse is
60/min, and respirations are 18/min. Examination shows weakness of the
proximal and distal muscles of the lower extremities. There is impaired
proprioception and vibratory sensation. Deep tendon reflexes are increased.
The gait is ataxic. The most likely diagnosis is
A. Guillain-Barre syndrome
B. Lambert-Eaton syndrome
C. myasthenia gravis
D. polymyositis
E. subacute combined degeneration of the spinal cord
Explanation:
The correct answer is E. This patient most likely has subacute combined
degeneration of the spinal cord, which is a condition caused by vitamin B12
deficiency. It is most often caused by pernicious anemia, but it may be
acquired in patients with strict vegetarian diets or small bowel disease. The
clinical manifestations include weakness, paraesthesias, loss of vibratory
sensation, increased deep tendon reflexes, and extensor plantar responses. The
gait is ataxic. Mental changes may also occur. The diagnosis is usually
established by measuring serum vitamin B12 levels. The treatment is vitamin
B12 replacement.
Guillain-Barre syndrome (choice A) is an acquired demyelinating neuropathy
that usually follows a viral respiratory infection or immunizations. It is
characterized by ascending weakness. Sensation is intact. A main diagnostic
clue is absent deep tendon reflexes.
Lambert-Eaton syndrome (choice B) is a neuromuscular disorder that causes
proximal muscle weakness, ptosis, and diplopia. Deep tendon reflexes are
depressed or absent. Repetitive nerve stimulation shows increased responses.
Small-cell carcinoma of the lung has been associated with this disorder.
Myasthenia gravis (choice C) is a neuromuscular disorder due to
autoantibodies to the acetylcholine receptors. It is characterized by weakness
and easy muscle fatigability. The extraocular and eyelid muscles are affected,
leading to diplopia and ptosis. Deep tendon reflexes are preserved.
Polymyositis (choice D) is a skeletal muscle disorder that is characterized by
progressive proximal muscle weakness. Patients often complain of difficulty
climbing stairs and brushing hair. Ocular muscles are generally not affected.
Creatine kinase is elevated. Muscle biopsy and electromyography confirm the
diagnosis.
A 61-year-old woman with chronic renal insufficiency due to long-standing
diabetes mellitus comes to the office with a fever, cough, shaking chills, and
fatigue. She has long-standing diabetes mellitus with her last hemoglobin A1C
being 9.1%, BUN 51 mg/dL, and creatinine 2.1 mg/dL. A chest radiograph
demonstrates a right lower lobe infiltrate. Oral antibiotics are prescribed for the
patient. The most correct statement concerning a diabetic patient with an
infectious process is:
A. Antimicrobial dosing must be adjusted due to decreased liver function
B. Antimicrobial dosing must be adjusted due to decreased renal function
C. Diabetics have the same epidemiology of pulmonary infections as nondiabetics
D. It is more common for diabetics to have lower lobe pneumonia when
compared to non-diabetics
E. Oral antibiotics are not efficacious for treating pneumonia in diabetics
Explanation:
The correct answer is B. All drugs (substances in general) are eliminated from
the body by means of either renal or hepatic clearance or both. This is one of
the most important concepts known by physicians. Almost every patient that
presents to inpatient medical services today has some impairment of one or
both of these mechanisms. This must be kept in mind when prescribing any
drug. In the case of this patient, renal antibiotic dosing schedules for variable
renal function are available to every student and house officer and should be
consulted regularly. This patient has a creatinine of 2.1 mg/dL, reflecting
abnormal renal function.
Antimicrobial dosing must be adjusted due to decreased liver function (choice
A), although true as a general rule, is not particularly relevant to diabetics since
their disease has no impact on liver function. We have no specific evidence
that this patient has impaired hepatic function.
Diabetics have the same epidemiology of pulmonary infections as nondiabetics (choice C) is incorrect as diabetics are more likely to suffer from
Staphylococcus aureus and fungal pneumonia.
It is more common for diabetics to have lower lobe pneumonia when compared
to non-diabetics (choice D) is incorrect. Although certain patient populations
tend to have anatomical variation in the location of their pneumonia, the
common relationship between all of them is that they suffer from some sort of
aspiration. Examples include intubated patients on mechanical ventilation and
alcoholics. Diabetics are not included in this group.
Oral antibiotics are not efficacious for treating pneumonia in diabetics (choice
E) is false. Oral antibiotic efficacy is determined by the causative organism and
location of the infection. Some specific dysfunctions such as gastric
achlorhydria or bowel obstruction may interfere with oral efficacy, but not
simply the presence of diabetes. Being a diabetic may make one prone to more
malignant infections. There is no data that initial treatment of community
acquired pneumonia in diabetics should be any different when compared to
non-diabetics.
An 11-year-old girl with insulin-dependent diabetes mellitus is brought to the
emergency department by a friend's father because of severe abdominal pain
and vomiting for the past 12 hours. The friend's father says that she has been
complaining of mild stomach "cramps" and "thirst" for the past few days, but
nothing this extreme. When he leaves the room, she reluctantly admits that she
has not been taking her insulin because she is mad at her parents for going
away to Europe and "leaving her" for 5 days. Her blood pressure is 100/70 mm
Hg, pulse is 98/min, and respirations are 30/min. Physical examination shows
dry skin and mucus membranes and diffuse abdominal pain. Laboratory studies
show:
Serum
Glucose
550 mg/dL
Sodium
138 mEq/L
Potassium
5.8 mEq/L
Chloride
94 mEq/L
Bicarbonate
10 mEq/L
Intravenous isotonic saline and insulin are given and she is admitted to the
pediatric intensive care unit for careful monitoring and management. Two
hours later, potassium is added, as her glucose and potassium levels begin to
drop rapidly. All seems to be going well until half an hour later when the nurse
runs over to you frantically saying that the patient suddenly complained of a
headache, began to vomit, and became "completely disoriented." You rush to
the bedside to find her obtunded. The most appropriate immediate management
is to
A. add more potassium to the intravenous solution
B. add phosphate to the intravenous solution
C. add sodium bicarbonate to the intravenous solution
D. administer a bolus infusion of mannitol
E. increase the dose of insulin
Explanation:
The correct answer is D. This patient has diabetic ketoacidosis (DKA) and has
most likely developed cerebral edema, which should be treated with mannitol
and dexamethasone. If this is not effective, hyperventilation should be induced.
DKA is characterized by hyperglycemia, osmotic diuresis, metabolic acidosis,
and elevated ketones. The typical clinical presentation is abdominal pain,
nausea, vomiting, polyuria, and lethargy. It is usually precipitated by an
infection, stress, a cessation of insulin intake, or surgery. The initial treatment
includes insulin and isotonic saline or Ringer's lactate. As glucose and
potassium levels begin to fall, potassium and glucose solutions must be given.
Bicarbonate is given to patients with severe acidosis. If the glucose level is
corrected too rapidly and glucose is not added to the intravenous fluids as the
levels fall below 300 mg/dL, cerebral edema may develop. Clinically, cerebral
edema manifests as acute altered mental status, vomiting, and a headache. A
CT scan is used to make the diagnosis.
Adding more potassium to the intravenous solution (choice A) would not be
the appropriate management of cerebral edema, which requires mannitol. This
patient's acute complication is inconsistent with hypokalemia, which is
typically associated with cardiac arrhythmias.
Adding phosphate to the intravenous solution (choice B) is an important part of
the treatment of DKA. However, this patient is most likely suffering from
cerebral edema, which is a life-threatening condition that requires rapid
treatment with mannitol, not phosphate. Patients with hypophosphatemia
typically complain of muscle weakness.
Sodium bicarbonate (choice C) is typically used in severely acidotic patients
with hypotension. It is not used to treat cerebral edema.
Increasing the dose of insulin (choice E) is incorrect because this patient has
cerebral edema, which is most likely caused by the rapid fall of glucose, and so
you would not want the glucose levels lowered even more rapidly.
A 41-year-old woman comes to the office because of a 40-pound weight gain
in the past 6 months. She is very upset because she has always been thin and
never had to watch her diet before. She has no change in appetite, no change in
eating habits, and has no other symptoms. She takes no medications, does not
drink alcohol, and smokes a pack of cigarettes a day for the past 10 years. She
rollerblades with her daughter every evening and tries to eat a well-balanced,
low-fat diet. She denies anxiety and any psychiatric problems. She tells you
that her mother and sister have always been overweight and they always
complain about their "apple-shaped" bodies. Her temperature is 37 C (98.6 F),
blood pressure is 120/80 mm Hg, pulse is 70/min, and respirations are 16/min.
Physical examination shows an obese abdomen and thin lower extremities, but
is otherwise unremarkable. The most appropriate next step is to
A. determine thyroid stimulating hormone levels
B. order a biochemical profile
C. order a dexamethasone suppression test
D. refer her to a nutritionist
E. schedule a CT scan of the abdomen
Explanation:
The correct answer is B. The initial work-up for weight gain is a detailed
history, including medication and drug use and recent smoking cessation, and a
biochemical profile. A biochemical profile may indicate the presence of
diabetes mellitus or Cushing syndrome. Diabetes will most likely lead to an
elevated glucose, while Cushing is sometimes associated with hypokalemia,
hypochloremia, metabolic alkalosis, hyperglycemia, and hypercholesterolemia.
An individual with an "apple-shaped" body typically has a large abdomen and
chest and thin legs. A "pear-shaped" body typically refers to a thin torso with
larger hips, buttocks, and legs. This is included in the vignette to show that the
patient's "obese abdomen" is consistent with her family's body type and is not
necessarily associated with Cushings or any other conditions.
Determining thyroid stimulating hormone levels (choice A) is appropriate in
the evaluation of weight gain if there is a negative drug history, no recent
smoking cessation, and a normal biochemical profile. Since hypothyroidism is
part of the differential diagnosis for weight gain, you should consider it early
in the work-up, but a biochemical profile should be performed first.
A dexamethasone suppression test (choice C) is part of the initial evaluation of
Cushing syndrome. You should first order a biochemical profile to determine if
this diagnosis is more likely than diabetes mellitus.
Referring her to a nutritionist (choice D) is inappropriate at this time. The case
says that the patient has had weight gain with no change in appetite or eating
habits. She requires evaluation for a medical condition, such as Cushing
syndrome, diabetes mellitus, thyroid disease, (even though this is usually
associated with a poor appetite), and fluid overload.
A CT scan of the abdomen (choice E) may be used to visualize an adrenal
mass leading to Cushing syndrome and weight gain. It may be ordered after a
biochemical profile, dexamethasone suppression test, and plasma cortisol. This
study is not appropriate at this time.
A 45-year-old woman returns to your office to discuss the results of "blood tests" drawn
3 days earlier when she was complaining of fatigue, weight loss, frequent urination, and
blurred vision for the last several weeks. She is a non-smoker and drinks 1 glass of wine
per week. She had no past medical or surgical history and a complete physical
examination 3 days earlier was unremarkable including a blood pressure of 130/72 mm
Hg. Laboratory studies show:
Based on these studies, you decide to begin treatment with rosiglitazone. At this time,
the most important additional study to perform is
A. alanine aminotransferase and aspartate aminotransferase
B. electrocardiogram
C. free thyroxine and T3 reuptake
D. 1-hour glucose tolerance test
E. 24 hour urine protein collection
Explanation:
The correct answer is A. This patient has type II (adult-onset) diabetes, which has been
diagnosed by a fasting glucose over 200 mg/dl and classic symptoms, including weight
loss, blurred vision, and polyuria. The class of drugs known as thiazolidinediones, which
includes rosiglitazone (Avandia) and pioglitazone (Actos) are often used as initial
monotherapy. The mechanism of action is through enhancing insulin sensitivity of
peripheral tissues. However, these medications have been shown to cause liver toxicity.
Therefore, it is recommended to check liver enzymes before initiating therapy and every
other month for the first year of therapy.
EKG (choice B) is not indicated as she has no cardiac complaints, no cardiac history,
and is normotensive.
Given her complaints of fatigue and weight loss, a thyroid disorder was a possibility in
your workup. However, given her normal TSH, further thyroid studies (choice C) are not
necessary at this time.
1-hour glucose tolerance test(choice D), is no longer used as a method for diagnosing
diabetes except as a screening test during pregnancy. The patient's fasting glucose of
206, along with her classic symptoms are enough to make the diagnosis.
It is important to screen all diabetic patients for renal disease. Though screening for
microalbuminuria is indicated, it is best performed by a spot urine
microalbumin/creatinine ratio, instead of a 24 hour urine test(choice E).
A 51-year-old man comes to the clinic for a pre-employment examination. He
has diet-controlled diabetes mellitus for 25 years, hypertension, glaucoma,
mild peripheral vascular disease, and osteoarthritis. His medications include
lisinopril, atenolol, aspirin, and acetaminophen as needed for pain. His blood
pressure is 160/80 mm Hg and pulse is 61/min. His examination is notable for
a left carotid bruit, a 2/6 systolic ejection murmur heard best at the left sternal
border, and clear lungs. His abdomen is soft with no masses but there is a
previously appreciated abdominal bruit. He hands you a piece of paper that
shows that his last BUN and creatinine were 65 mg/dL and 1.6 mg/dL
respectively. His last HA1C value was 8.3%. The most accurate statement
concerning this patient's condition is:
A. He has normal renal function
B. He has poorly controlled diabetes and severe diabetic renal disease
C. He has renal insufficiency
D. He is not a candidate for a renal transplant
E. He requires hemodialysis
Explanation:
The correct answer is C. A BUN and creatinine as high as this patient's reflect
a glomerular filtration rate that is below normal, therefore indicating renal
insufficiency.
By definition this is renal insufficiency and not normal renal function (choice
A).
Although this patient has a HA1C level of 8.3% reflecting a mean blood
glucose concentration of greater that 225 mg/dL and therefore poorly
controlled diabetes, he does not have severe diabetic renal disease (choice B)
based upon the fact that he has only mildly elevated renal indices.
The evaluation of patient as a transplant candidate is a complex process
involving medical, psychiatric, and social issues and since we have no
information on most of these points, it is not possible to say that this patient is
not a candidate for a renal transplant (choice D).
There are five indications for hemodialysis: uremia, hyperkalemia, acidosis,
volume, and uremic pericarditis. Since this patient has none of these he does
not require hemodialysis (choice E).
A 26-year-old man comes to the office for a periodic health maintenance
examination. He has no complaints at this time and does not take any
medications. His temperature is 37 C (98.6 F), blood pressure is 110/70 mm
Hg, pulse is 70/min, and respirations are 12/min. Physical examination reveals
a single, firm nodule in the left lobe of the thyroid gland. It is fixed and placed
with swallowing. The remainder of his examination is normal. Radioactive
iodine thyroid scintiscanning reveals that the nodule is "cold". Thyroid
function tests show TSH 1.14 mU/mL, T3 134 nmol/L, thyroxine 8 nmol/L.
The most appropriate next step is to
A. advise the patient to return in 3 months to evaluate the nodule for change
in size
B. begin 6 months of suppressive therapy with thyroxine
C. begin therapy with propylthiouracil
D. order a thyroid ultrasound in 3 months to evaluate for additional nodules
E. perform a fine needle aspiration of the nodule
Explanation:
The correct answer is E. This patient presented with a thyroid nodule that is
suspicious for malignancy. Risk factors for thyroid cancer include radiation
therapy to the neck in childhood and family history of thyroid cancer. The fact
that the nodule is firm and fixed in place is suggestive of malignancy. Thyroid
malignancies are more common in men than in women and more nodules in
young patients are malignant than in older patients. Radioactive iodine thyroid
scintiscanning is a test which tells us which areas of the thyroid gland are more
active and which are less active. Most thyroid malignancies are less active (i.e.,
cold). The appropriate management of this patient is a fine needle aspiration of
the nodule. This can be done safely in experienced hands and has very few
complications. It is of great benefit to the physician because it provides tissue
to make a definitive diagnosis and guide treatment.
Serial neck exams (choice A) for change in size of the nodule is inappropriate
because thyroid cancer found in its early stages is often curable.
Thyroxine therapy (choice B) is important in the management of thyroid
cancer and is often used after the thyroid cancer is removed with surgery.
Since TSH can stimulate cancers to grow more rapidly it is beneficial to
suppress TSH after someone is diagnosed with a thyroid cancer. The dose of
Levothyroxine used is the highest dose possible to suppress TSH without
causing the patient to be symptomatically hyperthyroid.
Propylthiouracil (choice C) inhibits the oxidation of iodide thereby decreasing
the production of thyroid hormone and inhibiting peripheral conversion of T3
to T4 in the blood. It is used in the management of Graves disease as well as
other hyperthyroid states. It is not useful in the management of a thyroid
nodule.
Thyroid ultrasound (choice D) is sometimes valuable in the work-up of a
thyroid nodule. An ultrasound can detect small nodules that even very
experienced hands cannot palpate. However these small nodules rarely have
malignant potential and a biopsy of these nodules hasn't been shown to
improve mortality. Also, there is no reason to wait 3 months for this
evaluation. A FNA should be performed with or without an ultrasound at this
visit or as close to this time as possible, not in 3 months.
A 52-year-old man comes to the clinic because he has been feeling weak and
dizzy for the past several days. He also tells you that at the same time he feels
somewhat restless and has had a mild headache. His past medical history is
significant only for some mild chronic low back pain for which he occasionally
takes acetaminophen. His temperature is 37.0 C (98.6 F), blood pressure is
128/78 mm Hg, pulse is 78/min, and respirations are 18/min. Physical
examination shows diffuse hyporeflexia and scant basilar crackles in the lungs.
Laboratory studies show a leukocyte count of 8,900mm3, hematocrit 40%,
platelets 295,000mm3, sodium 126 mEq/L, potassium 3.8 mEq/L, bicarbonate
18 mEq/L, blood urea nitrogen 9 mg/dL, creatinine 0.6 mg/dL, glucose 115
mg/dL, serum osmolality is 258 mOsm/kg (normal 280), and urine osmalality
is 150 mmol/L. A chest x-ray shows a 4-centimeter right upper lobe mass and
mediastinal adenopathy. The most appropriate next step in management is to
A. admit the patient to the hospital and start an infusion of hypertonic (3%)
saline until serum sodium normalizes
B. prescribe demeclocycline 150 mg orally 4 times per day, then follow up
in 2 weeks with an oncologist
C. refer the patient to cardiothoracic surgery for a lung biopsy
D. restrict the patient to 1500 mL of water per day and obtain CT scan of
the chest
E. schedule the patient for the next available oncology appointment
Explanation:
The correct answer is D. This patient is presenting with a classic syndrome of
inappropriate antidiuretic hormone secretion (SIADH). He has hyponatremia
with elevated urine osmolality in the presence of decreased serum osmolality.
The clinical symptoms are often vague like his, until more serious seizures or
coma present. One of the most common causes of SIADH is small cell cancer
of the lung, which he most likely has. The mainstay of treatment is with fluid
restriction once the serum sodium level is greater than 125 mEq/L. If the
patient's initial sodium is less than 125 mEq/L, then treatment with 0.9%
normal saline or possibly even more hypertonic solutions (choice A) may be
necessary. It is important not to correct the sodium level too quickly or a
deadly syndrome known as central pontine myelinolysis may result. The actual
appropriate rate is controversial. However, some recommend a rate of less than
0.5 mmol/L/hr. A CT scan of the chest will also be helpful to both the surgeon
and oncologist.
Demeclocycline is a medication that actually has nephrogenic diabetes
insipidus (the opposite of SIADH) as one of its side effects by blocking the
action of antidiuretic hormone. This medication is often adjunctive in the
treatment of SIADH. However, you would not simply want to follow up with
the patient in 2 weeks (choice B). They need much more vigilant care, as
further hyponatremia can result in severe neurologic sequellae.
Similarly, a lung biopsy (choice C) and a referral to an oncologist (choice E)
will certainly be eventually required in this patient. However, treating the
hyponatremia is of prime importance.
A 32-year-old woman comes to the office because of palpitations and anxiety
for a few months. She further complains that, "My right eye is bulging out of
my head and I see double." Examination reveals visual acuity of 20/20 in both
eyes, exophthalmos of the right eye, and redness in both eyes, worse in the left
eye. You order a sensitive thyroid stimulating hormone test, which comes back
at 0.1 U/mL. At this time the most correct statement about this patient's
condition is:
A. Cigarette smoking has been shown to increase the severity of
exophthalmos in this disease
B. Diplopia observed in this condition is from transient cranial nerve palsies
C. Exophthalmos and lid retraction improves as the patient's thyroid
hormone levels are appropriately treated
D. Ocular dryness, irritation, and redness are commonly observed in this
condition, but visual field deficits and blindness have not been reported
E. Unilateral exophthalmos is a rare finding in this disease
Explanation:
The correct answer is A. This patient has Graves disease or thyroid
ophthalmopathy. Cigarette smoking has been linked to an increased severity of
thyroid ophthalmopathy. The exact reason for this is not known, but patients
who smoke should be informed of this association as further encouragement to
quit smoking.
The diplopia in Graves disease is from infiltration of the extraocular muscles
causing restriction of extraocular motility and not from cranial nerve palsies
(choice B).
The degrees of exophthalmos and thyroid function run different clinical
courses. Treatment of thyroid dysfunction is certainly indicated, but this will
not affect the patient's ocular findings (choice C). Furthermore, the ocular
findings of Graves disease may precede the onset of dysthyroidism or become
apparent many months after the thyroid function has been treated.
Ocular dryness, redness, irritation, and diplopia are all very common signs and
symptoms of Graves disease; however, the clinician should be aware that
compression of the optic nerve can occur. If unrecognized and not treated, then
an optic neuropathy can cause visual loss (choice D). All patients with thyroid
ophthalmopathy should be referred to an ophthalmologist for evaluation.
Unilateral exophthalmos is not uncommon (choice E) and should not eliminate
the diagnosis of Graves disease. In fact, the most common cause of unilateral
exophthalmos is Graves disease.
A 17-year-old high school student is admitted to the hospital for treatment of
anxiety disorder when he reports acute loss of vision in the left eye. His past
medical history is significant for intractable anxiety disorder for the past 3
years. Physical examination reveals bilateral proptosis. There is no vision in
the left eye and vision in the right eye is normal. Extraocular movements are
intact bilaterally. The sinuses and ears are normal on exam. An MRI of the
brain is urgently performed and is normal. Recent laboratory studies show:
The next step is
A. buspirone administration
B. coumadin administration
C. diazepam administration
D. EEG
E. funduscopic examination
F. heparin administration
Explanation:
The correct answer is E. This patient's symptoms of anxiety, bilateral
proptosis, and a decreased thyroid stimulating hormone (TSH) are consistent
with hyperthyroidism with thyroid ophthalmopathy. A thorough funduscopic
exam will reveal papilledema. An uncommon complication of thyroid
ophthalmopathy (5% of cases) is optic neuropathy from edema of the
extraocular muscles with secondary compression of the optic nerve.
Anxiolytic therapy with buspirone (choice A) may be necessary, but treatment
of the underlying hyperthyroidism is the primary clinical concern. The next
most appropriate step is to perform a thorough funduscopic examination to
evaluate for papilledema.
Anticoagulation (choice B) is not necessary. This patient is suffering from
optic nerve compression secondary to extraocular muscle edema, not a stroke.
Anxiolytic therapy with diazepam (choice C) may be necessary, but treatment
of the underlying hyperthyroidism is the primary clinical concern.
Vision loss is a rare complication of a seizure which could be evaluated by
EEG (choice D). Seizure as a cause of vision loss is an unlikely possibility
given that this patient has anxiety, bilateral proptosis, and an elevated thyroid
stimulating hormone (TSH). These findings are all consistent with
hyperthyroidism and secondary thyroid ophthalmopathy.
Anticoagulation with heparin (choice F) is not necessary. This patient is
suffering from optic nerve compression secondary to extraocular muscle
edema, not a stroke.
A 65-year-old woman comes to the clinic for a follow up visit after being
diagnosed with type II diabetes mellitus. She is obese with a history of
hypertension, hyperlipidemia, and osteoarthritis. Despite a 3-month trial of diet
and exercise, her weight has increased by 3 pounds. In addition, her
hemoglobin A1C has increased from 7.8% to 9.0% and her fasting blood sugar
ranged from 167-188 mg/dL on the glucometer she now uses at home. With
the exception of an elevated glucose, her laboratory results are within normal
limits. The most appropriate pharmacotherapy for this patient is
A. Acarbose
B. Chlorpropamide
C. insulin NPH at bedtime
D. insulin 70/30 BID
E. metformin
Explanation:
The correct answer is E. Metformin acts predominately to decrease hepatic
glucose production in the liver. It is an excellent choice for obese patients with
diabetes because it promotes modest weight loss by actually decreasing the
amount of insulin necessary because of decreased glucose produced by the
liver. It is an appropriate drug for monotherapy or combination therapy of
diabetes. It should not be used in patients with renal failure or those patients
who are 80 years of older (creatinine clearance decreases with age) because or
the risk of lactic acidosis. When metformin is used correctly, complications are
uncommon. Because the amount of insulin secreted is not increased, it does not
cause hypoglycemia.
Acarbose (choice A) is an alphaglucosidase inhibitor. It works by decreasing
the rate of glucose entry into the blood stream, thereby preventing some of the
post prandial hyperglycemia. Acarbose has been shown to give a slight
decrease in hemoglobin A1C concentrations. In clinical practice, these drugs
are poorly tolerated because of gastrointestinal side effects such as diarrhea,
bloating, and gas.
Chlorpropamide (choice B) is a first-generation sulfonylurea. It works (as do
all drugs in the sulfonylurea class) by binding to a receptor on the beta cells of
the pancreas and stimulating insulin secretion. This class of drugs can be used
as first-line treatment or can be used in combination with other medications.
Chlorpropamide is rarely used today because of its propensity to cause more
hypoglycemia than newer drugs in the class.
Patients on multiple medications who continue to have poor glycemic control
can be tried on a bedtime dose of NPH insulin. (choice C). If this is not
effective, then twice a day injections with 70/30 should be added.
Insulin 70/30 (choice D) is typically used only if poor control is achieved with
oral agents. Obviously, most patients would prefer to take medication by
mouth rather than injecting themselves with insulin. An exception to this
would be a type II diabetic who presents with symptoms of hyperglycemia
such as weight loss, polyuria or polydipsia. These patients can be started on
insulin as initial therapy.
Above imformation from Clinical Geriatrics "Improving Disease Management
with New Treatments for Type 2 Diabetes Mellitus" Volume 8, number 7, June
2000.
A 54-year-old woman comes to the office complaining of "sluggishness",
decreased concentration, depression, and weight gain of 11 pounds over the
last 7 months despite dieting and exercising 3 days a week. Her temperature is
37 C (98.6 F), blood pressure is 110/70 mm Hg, pulse is 70/min, and
respirations are 12/min. Physical examination reveals a diffuse goiter, dry skin,
and a slightly hoarse voice. The remainder of her physical exam is normal with
the exception of "hung up" ankle jerk reflexes bilaterally. Laboratory studies
show:
The most appropriate next step in management is to
A. begin therapy with fluoxetine
B. begin therapy with levothyroxine
C. begin therapy with lovastatin
D. begin therapy with propylthiouracil (PTU)
E. begin therapy with vitamin B12 and folate
Explanation:
The correct answer is B. This patient has subclinical hypothyroidism (elevated
TSH with a normal free T4). Treatment is recommended if the patient has
symptoms. Ordering further studies such as thyroid function test or
thyroperoxidase antibody will help support your diagnosis. Clues to the
diagnosis of hypothyroidism include weight gain, difficulty concentrating,
hoarse voice, dry skin, thinning of the eye brows laterally, goiter, etc.
Treatment with levothyroxine should be initiated in our patient since she has
many of the classic symptoms of hypothyroidism. It is likely that this patient, if
untreated, would develop clinical hypothyroidism in the future, which would
require treatment. In a young, otherwise healthy person, therapeutic doses of
levothyroxine can be started initially. In older patients or patients with heart
disease low-dose levothyroxine should be started with a slow titration to
therapeutic doses to avoid cardiac complications.
Depression is often a part of hypothyroidism and the two disorders can often
mimic each other. Prior to starting a patient on an SSRI (choice A), reversible
causes of depression (such as hypothyroidism) need to be excluded. Since
depression is so prevalent, it is reasonable to reevaluate this patient for
depression after she is euthyroid.
Therapy with a statin drug (choice C) is not appropriate at this time. This
patient does have an elevated LDL and total cholesterol but two things need to
be remembered: first, hypothyroidism can cause high cholesterol so it is
necessary to recheck the patient's cholesterol when euthyroid. Second, this
patient needs a trial of exercise and diet prior to being placed on a lipidlowering drug.
Propylthiouracil (choice D) is a therapy for hyperthyroidism, not
hypothyroidism. It acts by inhibiting the iodination of thyroid hormone and
acts to block the peripheral conversion of T3 to T4.
This patient has a macrocytic anemia, which is often associated with B12 and
folate deficiency (choice E). A less common cause of macrocytic anemia is
hypothyroidism. This patient's anemia may resolve once treatment with
levothyroxine is initiated.
A 54-year-old woman comes to the office complaining of "sluggishness",
decreased concentration, depression, and weight gain of 11 pounds over the
last 7 months despite dieting and exercising 3 days a week. Her temperature is
37 C (98.6 F), blood pressure is 110/70 mm Hg, pulse is 70/min, and
respirations are 12/min. Physical examination reveals a diffuse goiter, dry skin,
and a slightly hoarse voice. The remainder of her physical exam is normal with
the exception of "hung up" ankle jerk reflexes bilaterally. Laboratory studies
show:
The most appropriate next step in management is to
A. begin therapy with fluoxetine
B. begin therapy with levothyroxine
C. begin therapy with lovastatin
D. begin therapy with propylthiouracil (PTU)
E. begin therapy with vitamin B12 and folate
Explanation:
The correct answer is B. This patient has subclinical hypothyroidism (elevated
TSH with a normal free T4). Treatment is recommended if the patient has
symptoms. Ordering further studies such as thyroid function test or
thyroperoxidase antibody will help support your diagnosis. Clues to the
diagnosis of hypothyroidism include weight gain, difficulty concentrating,
hoarse voice, dry skin, thinning of the eye brows laterally, goiter, etc.
Treatment with levothyroxine should be initiated in our patient since she has
many of the classic symptoms of hypothyroidism. It is likely that this patient, if
untreated, would develop clinical hypothyroidism in the future, which would
require treatment. In a young, otherwise healthy person, therapeutic doses of
levothyroxine can be started initially. In older patients or patients with heart
disease low-dose levothyroxine should be started with a slow titration to
therapeutic doses to avoid cardiac complications.
Depression is often a part of hypothyroidism and the two disorders can often
mimic each other. Prior to starting a patient on an SSRI (choice A), reversible
causes of depression (such as hypothyroidism) need to be excluded. Since
depression is so prevalent, it is reasonable to reevaluate this patient for
depression after she is euthyroid.
Therapy with a statin drug (choice C) is not appropriate at this time. This
patient does have an elevated LDL and total cholesterol but two things need to
be remembered: first, hypothyroidism can cause high cholesterol so it is
necessary to recheck the patient's cholesterol when euthyroid. Second, this
patient needs a trial of exercise and diet prior to being placed on a lipidlowering drug.
Propylthiouracil (choice D) is a therapy for hyperthyroidism, not
hypothyroidism. It acts by inhibiting the iodination of thyroid hormone and
acts to block the peripheral conversion of T3 to T4.
This patient has a macrocytic anemia, which is often associated with B12 and
folate deficiency (choice E). A less common cause of macrocytic anemia is
hypothyroidism. This patient's anemia may resolve once treatment with
levothyroxine is initiated.
A 83-year-old man with a history of obesity, diabetes mellitus type 2,
hypertension, chronic renal insufficiency, hyperlipidemia, and coronary artery
disease comes to your office after being awakened from sleep by severe pain in
his "right first toe." He says that he had a sudden onset of acute pain in the toe
rapidly followed by erythema, swelling, tenderness, and warmth. His
temperature is 37.0 C (98.6F), blood pressure is 170/60 mmHg, pulse is
97/min, and respirations are 19/min. Physical examination is normal except for
swelling and severe tenderness over the metatarsophalangeal joint on the right
foot. At this time the most accurate statement about this patient's condition is:
A. Allopurinol would be a reasonable therapy now by decreasing his serum
uric acid levels
B. Decreasing his dietary protein may have some role in decreasing
frequency of recurrent attacks
C. Elevated serum urate levels in the past should have been treated to
decrease the likelihood of attacks
D. Most patients will only experience a single attack in their lifetime
E. Overproduction of uric acid is the etiology of these attacks in most
patients.
Explanation:
The correct answer is B. This patient has gout. Pain in the MTP joint of his
first toe is a classic symptom and is known as podagra. All of the comorbid
medical conditions associated with gout are listed in the case and include
diabetes, hypertension, coronary artery disease, hyperlipidemia, obesity, and
chronic renal disease. Excessive dietary purine intake along with alcohol may
also contribute. Patients with gout should be encouraged to modify their diets
to minimize secondary causes of hyperuricemia.
Allopurinol (choice A) is a xanthine oxidase inhibitor which is an effective
therapy for hyperuricemia. Allopurinol has no role in the treatment of acute
gout since it may cause the mobilization of tophi, thereby causing increased
pain. It is useful to keep the uric acid level within normal limits. NSAIDS are
the treatment of choice for acute gout. Colchicine and steroids are other
alternatives.
Asymptomatic hyperuricemia (choice C) without evidence of gout is not
typically treated. Most physicians would wait until the first attack of gout
before initiating allopurinol treatment. Of note, uric acid levels are not helpful
in the diagnosis of acute gout because they can be normal in some patients.
Most patients have recurrences of gout in their lifetime. The majority of
patients even have recurrences within the first year. Therefore a single attack,
(choice D), is incorrect.
Overproduction of uric acid (choice E) is responsible for about 10% of the
patients with gout. This can be a primary process secondary to defects in the
purine enzyme pathways or can be secondary from alcohol, heme malignancy,
or chemotherapy. 90% of the patients with gout are under-excreters, which
means they excrete less than 700 mg of uric acid per day.
A 35-year-old woman comes to the office because of tremors, weakness,
weight loss despite an increased appetite, frequent bowel movements, and
"itchy legs." She does not drink alcohol or caffeine and she does not smoke
cigarettes. She appears anxious and fidgety, and has a "frightened" look. Her
blood pressure is 120/80 mm Hg and pulse is 88/min. Physical examination
shows a proptosis with stare and lid lag. The thyroid gland is lobular and
asymmetrically enlarged. There are waxy, infiltrated plaques over the dorsum
of her legs and the affected area has a peau d'orange appearance. An ECG
shows sinus tachycardia. Laboratory studies show:
Treatment options are discussed, and she decides upon radioactive iodine. In
discussing radioactive iodine therapy with the patient, you should inform her
that
A. continuous or repeated doses of radioiodine are usually necessary
B. hepatitis, arthralgias, agranulocytosis, and a rash are common side
effects of radioiodine therapy
C. patients treated with radioiodine will often require replacement doses of
thyroid hormone within 10 years
D. radioiodine therapy is the treatment of choice if she is currently pregnant
E. there is plenty of evidence that the dose of radioiodine that she will
receive has leukemogenic effects
Explanation:
The correct answer is C. This patient has Grave's disease, which is a common
cause of hyperthyroidism, and it important to discuss the advantages and
disadvantages of the treatment options. The major disadvantage of radioiodine
therapy is that 40-70% of patients will develop hypothyroidism and require
thyroid hormone replacement within 10 years. Grave's disease is characterized
by diffuse goiter, dermopathy, and ophthalmopathy. The clinical
manifestations are that of hyperthyroidism and include palpitations, tremors,
weakness, weight loss despite an increased appetite, frequent bowel
movements, heat intolerance, and oligomenorrhea or amenorrhea. Proptosis,
lid lag, and stare are often present. The dermopathy, which is called pretibial
myxedema, is usually characterized by waxy, infiltrated plaques over the
dorsum of the legs and a peau d'orange appearance. Laboratory studies show
low or undetectable levels of TSH, and elevated levels of T4, T3, and the
RAIU and RT3U. Radioactive iodine therapy is a good choice for treating
hyperthyroidism in adult patients, patients with previous thyroid surgery, and
those who cannot undergo surgery. Long-term antithyroid therapy with
methimazole or propylthiouracil is usually used in children.
It would be incorrect to inform her that continuous or repeated doses of
radioiodine is usually necessary (choice A). Continuous or repeated therapy
with antithyroid drugs (methimazole or propylthiouracil) is often necessary.
It would be incorrect to inform her that hepatitis, arthralgias, agranulocytosis,
and a rash are common side effects of radioiodine therapy (choice B) because
these are associated with antithyroid drugs, not radioiodine therapy.
It would be incorrect to inform her that radioiodine therapy is the treatment of
choice if she is currently pregnant (choice D) because radioiodine therapy
should never be given to pregnant women. Antithyroid drugs are usually used
during pregnancy.
It would be incorrect to inform her that there is plenty of evidence that the dose
of radioiodine that she will receive has leukemogenic effects (choice E)
because this is not true. There is no evidence to support this statement (in
adults).
A 61-year-old man with angina, hypertension, hypercholesterolemia, and
peptic ulcer disease comes to the clinic with a recurrent right great toe pain that
has been severe for the past 2 days. His medications include atenolol,
lovastatin, famotidine, and an occasional aspirin. Physical examination shows
an obese man with an edematous, erythematous hallux metatarsophalangeal
joint. Examination is otherwise unremarkable. X-ray of the right great toe
demonstrates an edematous hallux metatarsophalangeal joint. There is no
fracture or bony erosions. The most appropriate pharmacotherapy at this time
is
A. acetaminophen
B. allopurinol
C. colchicine
D. indomethacin
E. prednisone
Explanation:
The correct choice is C. This patient is presenting with signs and symptoms of
gout. Gout is a crystal-induced arthropathy that often presents in older, obese
men. Over time, crystal deposits in the joints cause an erosive arthropathy.
Colchicine may be used as an alternative to nonsteroidal antiinflammatory
medications (NSAID) when there is a history of peptic ulcer disease, excessive
bleeding, or intolerance to NSAIDs. Colchicine may also be used as a
prophylactic treatment for gout.
Acetaminophen (choice A) is not indicated for an acute gout attack because of
its lack of antiinflammatory activity.
Allopurinol (choice B) is used to prevent gouty attacks. It serves no role in an
acute attack.
Indomethacin (choice D) is a type of nonsteroidal antiinflammatory medication
(NSAID) that is relatively contraindicated given the patient's history of peptic
ulcer disease. However, indomethacin is often considered one of the typical
first-line agents for acute gout attacks.
Oral steroids (choice E) are not typically used for an acute gouty attack.
Intravenous glucocorticoids are sometimes considered for patients who cannot
tolerate oral medications or others who have not responded to other therapies.
Intraarticular methyl prednisone or betamethasone is generally reserved for
patients with acute gouty monoarticular disease. Steroids are used extensively
to treat other inflammatory arthritides like rheumatoid arthritis.
A 23-year-old man is admitted to the medical services for dehydration. He had
just completed a marathon that afternoon and was brought to the hospital by
his sister who found him to be lethargic and confused. His sister informs you
that he has been training very vigorously for the marathon and completed the
marathon in near-record time by not stopping for rehydration at all of the
available rest stops. On examination, the patient is a well-developed man. He is
speaking using unclear words and is warm to the touch with stable vital signs.
His skin is very dry and his lips are chafed. His serum sodium is 163 mEq/L.
The result is confirmed with the laboratory. The most appropriate management
at this time is
A. intravenous half normal saline repletion
B. intravenous lactated ringers repletion
C. intravenous normal saline repletion
D. oral free water repletion
E. oral thiazide diuretics
Explanation:
The correct answer is D. This patient has hypernatremia as a consequence of
insensible free water losses. He needs free water repletion with one half the
free water deficit being given in the first 12 hours and the remaining half over
the next 24 hours. The serum sodium should fall by no more than
0.5mEq/L/hour (12 mEq/day).
Intravenous half-normal saline repletion (choice A) will worsen the condition.
Although there is more free water in this preparation, the added sodium will
likely worsen this patient's condition.
There is no role for intravenous lactated ringers repletion (choice B) in
correcting hypernatremia since it has a similar sodium content as normal
saline.
Intravenous normal saline repletion (choice C) will aggravate the condition by
providing only some free water and the remainder sodium. This is used to
replete some forms of hyponatremia.
Oral thiazide diuretics (choice E) are often a treatment for hypernatremia when
salt sodium excess is the primary problem. This can occur with hypertonic
dialysis solutions.
An 18-year-old man with type I diabetes mellitus is brought to the emergency
department by a friend after being found comatose. There is a known history of
noncompliance with medications, however, there is no known history of drug
use. Vital signs are: temperature 37 C (98.6 F), blood pressure 80/65 mm Hg,
pulse 110/min, and respirations 17/min. Oxygen saturation obtained while the
patient is receiving supplemental oxygen of 2 L/min via nasal cannula is 98%.
The patient is comatose and is taking rapid, shallow breaths. Deep tendon
reflexes are hypoactive. An intravenous line has been placed in the field. A
fingerstick glucose is 430 mg/dL. An arterial blood gas, basic chemistry panel,
and toxicology screen has been sent to the laboratory. The next step in the
management of this patient is
A. a chest x-ray
B. an endotracheal intubation
C. an intravenous fluid replacement with insulin
D. methadone
E. a pulmonary artery catheter insertion
Explanation:
The correct answer is C. This patient is suffering from diabetic ketoacidosis
(DKA) caused by a severe deficiency of insulin. Clinical symptoms include
coma, rapid and shallow breathing, high serum glucose levels, and metabolic
acidosis. The immediate management of this patient includes intravenous fluid
replacement and insulin infusion. When laboratory results return, electrolyte
imbalances must also be corrected.
A chest x-ray (choice A) would be complementary to a complete the
evaluation of any comatose patient. In this patient with a picture of diabetic
ketoacidosis, a chest x-ray would be a secondary concern. The first priority is
intravenous fluid replacement and insulin therapy.
Endotracheal intubation (choice B) is not necessary at this point as the patient
has a normal oxygen saturation. Adequacy of respiration will need to be
reassessed when the arterial blood gas results are available. The first priority is
intravenous fluid replacement and insulin therapy.
Methadone (choice D) is used to treat heroin dependency.
A pulmonary artery catheter (choice E) is not yet necessary as the patient is at
the present time hemodynamically stable. The first priority is intravenous fluid
replacement and insulin therapy.
A 46-year-old farmer is admitted to the hospital with confusion, profound
weakness, and hypotension. His primary care physician was contacted after
obtaining a history from his wife. One week prior to the presentation, the
patient saw his primary care physician with complaints of increasing fatigue
and weakness for the past few months. Thyroid function tests performed at that
time were reported as normal according to the primary care physician. Two
days prior to admission, the farmer's wife noticed that he was even weaker and
complained of dizziness on standing. He was tremulous and sweaty. His
symptoms improved by eating food. He attributed that to hunger and did not
report it to his primary care physician. One day prior to his admission to the
hospital, he developed flu-like symptoms with a low-grade fever. On the day
of admission, he experienced increasing confusion, profound weakness, and
inability to stand, because of lightheadedness. On examination, the patient is
drowsy, confused, and appears unwell. His temperature is 37.2 C (99 F), blood
pressure is 100/60 mm Hg supine, falling to 74/50 mm Hg when sitting up, and
his pulse is 108/min. He is pale with cool extremities and no skin
pigmentation. No other abnormalities are noted on physical examination. A
capillary blood glucose measurement is 64 mg/dL. Initial laboratory studies
show:
Appropriate initial management of this patient includes administration of
A. dextrose
B. dextrose and norepinephrine
C. dextrose, saline, and levothyroxine
D. saline, norepinephrine, and antibiotics
E. saline, steroids and antibiotics
Explanation:
The correct answer is E. This patient is dehydrated with postural hypotension
and circulatory failure. These findings are seen with adrenal insufficiency.
Biochemically, the patient has a low sodium concentration, high normal
potassium concentration, and an increased creatinine concentration. All
findings are consistent with loss of mineralocorticoid action. These are the
features of primary adrenal failure. This patient requires immediate
management of adrenal crisis with adequate hydration with saline and
administration with intravenous steroid. Relatively minor urinary or respiratory
tract infections can precipitate adrenal crisis and should be appropriately
treated.
The patient's hypoglycemia does not require specific management and will
correct itself without dextrose (choice A).
Dextrose is not indicated as stated above and norepinephrine (choice B) is not
indicated before adequate aggressive hydration for the correction of an adrenal
crisis.
Dextrose, saline, and levothyroxine (choice C) are incorrect, because dextrose
and levothyroxine are not indicated. The patient's hypoglycemia should correct
itself, and there is no indication for dextrose or levothyroxine in this patient.
Although saline and antibiotics are indicated in this patient, norepinephrine
(choice D) is not indicated in the initial management of this patient.
A 68-year-old woman is admitted to the hospital because of lethargy and
dehydration. She lives alone and has had regular checkups in the office. Her
past medical history is significant for diabetes mellitus, which is controlled
with diet and oral antidiabetic agents. She has a history of mild systolic
hypertension, treated with a thiazide diuretic. Her last visit to your office was 3
months ago, at which time she was started on digoxin for control of heart rate.
Since then, she has been taking digoxin and the diuretics without fail with
good control of her heart rate and no evidence of heart failure. On admission,
she is lethargic, but can be easily aroused. Her skin and mucous membranes
are dry. Her temperature is 37.8 C (100.0 F), blood pressure is 110/70 mm Hg,
and pulse is 90/min. Examination of the chest, abdomen, and extremities is
normal. An electrocardiogram shows atrial fibrillation. Her laboratory studies
show:
Urine analysis shows red cells, white cells, and few bacteria without any
protein. An abdominal radiograph reveals a small, calcified density in the
region of the right kidney. The most likely cause for this patient's metabolic
abnormality is
A. hyperparathyroidism
B. hyperthyroidism
C. metastatic malignancy
D. multiple myeloma
E. thiazide diuretic overdose
Explanation:
The correct answer is B. Hyperthyroidism may cause increased calcium by
stimulating bone resorption. Serum calcium levels normalize when the patient
becomes euthyroid. Patients with hyperthyroidism do have cardiac effects from
increased levels of thyroid hormone. Recent onset of atrial fibrillation, without
cardiac cause, should raise suspicion for hyperthyroidism and should be
investigated further.
Patients with hyperparathyroidism (choice A) typically have elevated serum
calcium and parathyroid hormone levels, normal or elevated urine calcium
excretions, and low or normal plasma concentration of phosphates.
Hypercalcemia of malignancy (choice C) is usually very high and seen in
patients with solid tumors, including lung carcinoma, breast carcinoma, and
squamous cell carcinoma of the head and neck. The hypercalcemia is thought
to be caused by parathyroid hormone related protein secreted by the tumor.
Patients with hematological malignancies may also have increased serum
calcium levels, but this is thought to be due to cytokines causing increased
osteoclastic activity in the bone.
Multiple myeloma (choice D) causes hypercalcemia by increased cellular lysis.
Urine analysis shows protein in urine in multiple myeloma patients.
Thiazide diuretic overdose, (choice E) may increase serum calcium levels, but
serum phosphate would likely be depressed. In this patient, although the
calcium levels are elevated, the phosphorus levels are high normal.
A 37-year-old accountant is hospitalized for a laparoscopic cholecystectomy.
The day after his surgery, he reports feeling palpitations in his chest. He says
that even prior to his hospitalization he had been feeling nervous and has
noticed himself perspiring more easily. His past medical history is significant
for a resection of a benign brain tumor during childhood. He also mentions that
he may have lost weight, although he has not been dieting. Physical
examination reveals a thin, anxious appearing male. His lungs are clear and
cardiac auscultation demonstrates an irregularly irregular rhythm and no
murmurs. Neurologic examination is significant for a fine tremor in both
hands. An electrocardiogram performed at the bedside shows atrial fibrillation.
The most appropriate study at this time to evaluate this patient's symptoms is
A. a chest x-ray
B. a CT scan of the head
C. an exercise tolerance test
D. a serum thyroid stimulating hormone
E. a ventilation/perfusion scan
Explanation:
The correct answer is D. Nervousness, tremor, heat intolerance, and weight
loss are classic signs and symptoms of hyperthyroidism. Hyperthyroidism is a
well known cause of atrial fibrillation. This arrhythmia will respond to the
treatment of the underlying endocrine abnormality. A serum thyroid
stimulating hormone (TSH) level will be abnormally low in patients with
hyperthyroidism and is a very specific test for this disorder.
A chest x-ray (choice A) is not a useful study in the evaluation of atrial
fibrillation in the setting of hyperthyroidism. There is no reason to suspect
other primary lung diseases in an otherwise healthy young male to warrant a
chest x-ray.
A CT of the head (choice B) will not provide information about the etiology of
the patient's atrial fibrillation. He has a distant history of surgical resection of a
benign brain tumor and there is no reason to suspect metastatic disease. There
is no association between primary brain pathology and atrial fibrillation.
An exercise tolerance test (choice C) is an examination to evaluate for cardiac
ischemia or past infarct. This study is performed by having a patient exercise
on a treadmill while wearing the electrocardiogram leads. A continuous EKG
is performed during exercise to monitor for signs of ischemia. Cardiac
ischemia is a very common cause of atrial fibrillation. However, there is no
reason to suspect an ischemic etiology of this arrhythmia in a young, otherwise
healthy patient.
A ventilation/perfusion scan (choice E) is a good diagnostic study for the
evaluation of a pulmonary embolism. A pulmonary embolism is a known cause
of atrial fibrillation. Although recent surgery and hospitalization does put the
patient at increased risk for thromboembolic disease, he does not have
shortness of breath, chest pain, or other classic symptoms of a pulmonary
embolism. Young and healthy patients do develop thromboembolic disease in
the setting of pelvic and lower extremity trauma, and prolonged hospitalization
or bedrest.
A 32-year-old woman has had severe right-sided flank pain for the past 2 days.
You have been taking care of her for the past year for rheumatoid arthritis. She
has been steroid dependant and recently started taking methotrexate as a
steroid-sparing agent. Her usual rheumatoid arthritis flares occur in her hands
and wrists, as well as a macular rash. Her blood pressure is 100/50 mm Hg
lying down and 80/40 sitting up. Her pulse is 120/min lying down and was
145/min sitting up. She looks very ill and fatigued. Physical examination
shows moderate right upper quadrant abdominal tenderness and severe right
sided flank tenderness. She is hydrated with lactated ringer's solution in the
office while her laboratory results are pending. After 2 liters of intravenous
fluids, the patient's blood pressure is 90/45 mm Hg lying down and she still
feels too weak to stand up. Laboratory studies show:
Urinalysis: >25 WBC, 5-10 WBC, + leukocyte esterase, + nitrate, + ketones,
no glucose
The most appropriate initial step in management is to
A. administer hydrocortisone, intravenously
B. call for an endocrinology consult
C. check serum cortisol level
D. infuse type O- blood
E. start antibiotics, intravenously
Explanation:
The correct answer is A. This patient is on chronic steroids for her rheumatoid
arthritis. She has a urinalysis consistent with a urinary tract infection and given
her flank tenderness and elevated WBC, probably has pyelonephritis.
However, given her chronic steroid use, this patient is relatively adrenal
insufficient and can't mount a stress response. She therefore has orthostatic
hypotension with eosinophilia and a low glucose. Additionally, her serum
sodium is low and her potassium is high, another useful clue to her
mineralocorticoid deficiency. If adrenal insufficiency is suspected, then stress
dose IV steroids would be the initial management
An endocrinology consult (choice B) would be too time consuming and would
just delay life saving treatment.
Checking serum cortisol levels (choice C) would be helpful to solidify the
diagnosis of adrenal insufficiency. However, IV steroids need not be delayed
given this patient's history and classic lab findings. Furthermore, this question
asks for the next step in management, not diagnosis.
This patient doesn't require a blood transfusion (choice D) for her anemia. It is
most likely her steroid deficiency that is contributing to her hypotension, not
her chronic anemia.
This patient will require IV antibiotics (choice E) for the pyelonephritis, but
the first initial step for management would include stress-dose steroids.
A 44-year-old obese woman comes to the clinic for a routine follow up visit
for diabetes. Her diabetes has been poorly controlled in the past year and her
hemoglobin A1c level was last at 11%. She denies any new problems with her
health. On physical examination, you note firm, non-pitting induration on her
upper back with a clear cut-off border. Within the indurated areas there are
small papules resembling follicular prominences. Under her breasts are beefyred patches in the moist areas with satellite lesions. On her left pretibial area,
there is a dusky-red elevated plaque with a sharply circumscribed border.
There is an orange-hue to this lesion, and the center of the lesion is flattened
and atrophic. In the order of these descriptions, the cutaneous manifestations of
diabetes that this patient has are
A. scleredema, candidiasis, necrobiosis lipoidal diabeticorum
B. scleredema, candidiasis, stasis dermatitis
C. scleredema, tinea corporis, stasis dermatitis
D. scleroderma, candidiasis, necrobiosis lipoidal diabeticorum
E. scleroderma, tinea corporis, stasis dermatitis
Explanation:
The correct answer is A. Scleredema, candidiasis, necrobiosis lipoidal
diabeticorum (NLD) is correct, because these three are all commonly
associated with diabetes. Scleredema is a connective disorder with sudden
onset of marked, nonpitting, symmetric induration of the posterior and lateral
aspects of the neck spreading to shoulders, upper back, and proximal arms. The
indurations are of wood-like consistency. Erythematous papular eruption
occurs during the early stage of the disease. A syndrome has been recognized
consisting of scleredema of long duration, obesity, maturity onset, latent or
overt diabetes, and high incidence of cardiovascular disease. Candida albicans
flourishes in the recesses created by redundant skin folds. Exacerbating factors
may include diabetes mellitus, systemic medications, nutritional factors, and
diminished salivary function. NLD is relatively asymptomatic, and is
characteristically found on the anterior and lateral lower legs. They may also
be present on the face, arms, and trunk. They begin as small, red nodules that
enlarge to a plaque with irregular, flattened and eventually depressed atrophy.
NLD seems to be a marker for diabetes.
Stasis dermatitis (choice B) is incorrect because classical stasis dermatitis
appears on the medial surface of lower extremities with hyperpigmented
patches mixed with occasional erythematous plaques. Often times, one can
appreciate tortuous enlarged varicose veins.
Tinea corporis (choice C) is incorrect because it generally presents with
erythematous annular plaques, with central clearing and no satellite lesions.
Scleroderma (choice D) and (choice E) is incorrect, because this is a
connective disorder associated with the tightening of the skin, difficulty
swallowing, telangiectasia, calcinosis, sclerodactyly, and other systemic
involvement.
A 45-year-old man comes to the clinic for a follow-up visit for hypercholesterolemia. On the previous visit,
him on a diet to lower his cholesterol. Now he tells you that he was following the diet, although he does not
same time he expresses concern that because his father died of a heart attack, dietary changes might not be s
him. He hands you an article that he downloaded from the Internet extolling the virtues of a new cholesterol
drug. You promise to look at the article and tell the patient what you think about it during his next visit in 3
article describes a double-blind clinical trial in which patients with cholesterol levels over 240 were assigned
groups: diet change only, drug only, or diet change and drug combined. Patients were followed over a 6-mon
and changes in cholesterol level from baseline was computed. The results of the study are presented in the ta
The drug was also shown to have significant side effects in 10% of the patients taking the drug. Based on thi
your recommendation to the patient should be
A. because there is no difference between the effects of dietary change or taking the drug, the patient can
which one he prefers
B. diet alone is sufficient and adding the drug will not provide enough clinical advantage to warrant the r
side effects
C. studies found on the Internet are not a good source of information for making treatment decisions
D. taking the drug by itself has sufficient clinical merit, so that dietary change is not required
E. taking the drug either with or without dietary change has sufficient clinical effect to merit giving the p
prescription
F. the drug should be taken, but only in conjunction with dietary change
G. the results of this study may not apply to this patient and a 3-month trial dosage of the drug should be
what effect it has
Explanation:
The correct answer is B. All three conditions show a lowering of cholesterol levels and the p-values indicate
significant differences among all three groups. However, the magnitude of the differences among the groups
these differences are not clinically significant. Changes of 1 or 2 points in cholesterol levels are unlikely to b
meaningful. Because dietary change provides a notable lowering of cholesterol levels and avoids the possibl
of the drug, this is the therapeutic option that gives the best benefit/risk ratio. Note that, of course, the physic
discuss with the patient his concerns that diet alone is not sufficient and explain the reasoning for this decisio
Because there is no difference between the effects of dietary change or taking the drug, the patient can choos
he prefers (choice A), taking the drug by itself has sufficient clinical merit, so that dietary change is not requ
D), taking the drug either with or without dietary change has sufficient clinical effect to merit giving the pati
prescription (choice E), and the drug should be taken, but only in conjunction with dietary change (choice F)
incorrect because the three groups are statistically significant; however, not clinically significant.
Studies found on the Internet are not a good source of information for making treatment decisions (choice C
because although everything on the Internet cannot be taken at face value, there are a number of excellent sit
publish valid, important medical information. The fact that the article was from the Internet, by itself, does n
the article as a source of information.
The results of this study may not apply to this patient and a 3-month trial dosage of the drug should be given
effect it has (choice G) is incorrect, because although the patient may have particularities which make the ef
different than the published study, the study results should be taken as accurate, unless and until an individu
experience proves differently. Otherwise, all research would be useless and every drug would have to be test
patient.
A 60-year-old man with diabetes mellitus, hypertension, hyperlipidemia, and
chronic renal insufficiency is admitted to the hospital because of
lightheadedness. His medications include NPH insulin, amlodipine, and
simvastatin. He is allergic to penicillin to which he gets an angioedema. His
temperature is 37.1 C (98.8 F), blood pressure is 98/65 mm Hg, pulse is
87/min, and his respiratory rate is 22/min. On exam, he is ill appearing. His
cardiac rhythm is regular and breath sounds are clear bilaterally. His
abdominal exam is benign. A chest radiograph shows clear lungs. An
electrocardiogram shows a sinus rhythm with peaked T waves. Laboratory
studies show a serum sodium of 134 mEq/L, glucose of 98 mg/dL, and
potassium of 6.2 mEq/L. The most appropriate intervention at this time is
A. administration of glucose, orally
B. administration of insulin and glucose, intravenously
C. administration of furosemide, orally
D. administration of ringers lactate, intravenously
E. administration of sodium chloride, intravenously
Explanation:
The correct answer is B. Hyperkalemia with electrocardiographic changes
(peaked T waves) requires immediate medical treatment to prevent the onset of
hyperkalemia-induced dysrhythmias. Administration of insulin intravenously
causes serum potassium to move intracellularly, acutely reducing serum
potassium. Glucose is co-administered to prevent insulin-induced
hypoglycemia.
Oral glucose (choice A) is of no utility in the treatment of hyperglycemia.
Even when insulin is used for the treatment of hyperkalemia, glucose should be
co-administered intravenously.
Oral furosemide (choice C) is an appropriate choice for the long-term
treatment of hyperkalemia, i.e., once the acute hyperkalemia has been treated.
Interventions such as insulin and glucose simply cause the potassium to shift
intracellularly. Its excretion can then be promoted by administration of diuretic
agents such as furosemide.
Ringers lactate (choice D) should be avoided with hyperkalemic patients since
it contains potassium as a constituent (4 mEq/L).
Sodium chloride (choice E) is not useful in the management of hyperkalemia.
A 37-year-old woman comes to the office because of a 3-month history of
"tiredness". She says that she has not felt like herself for "quite some time
now" but this fatigue is making it difficult to just get out of bed. She has been
calling in sick to work a couple of times and week and her supervisor
recommended that she "seek help." She has no other symptoms, does not have
any chronic medical conditions, does not take any medication, and has not
recently suffered a concussion. She denies severe psychological stress. Her
temperature is 37.2 C (99.0 F), blood pressure is 110/70 mm Hg, pulse is
65/min, and respirations are 15/min. Her physical examination is
unremarkable. A complete blood count is normal. The most appropriate next
step is to
A. order a biochemical profile
B. order a chest x-ray
C. prescribe fluoxetine
D. reassure her that this will pass and schedule a follow-up visit in 1 week
E. refer her to a psychiatrist
Explanation:
The correct answer is A. The initial evaluation of a patient with fatigue
includes a thorough history and physical examination, a complete blood count,
and a biochemical profile. These will evaluate the patient for causes of fatigue
such as anemia, uremia, diabetes mellitus, adrenal insufficiency, hypokalemia,
hyponatremia, and hepatitis. If these tests are negative, thyroid function tests
and the evaluation for an infection and an occult malignancy are indicated. If
all studies are negative, a nutritional deficiency, depression, a chronic viral
infection, or chronic fatigue syndrome should be considered. Even if chronic
fatigue syndrome is suspected, routine laboratory studies must be performed to
exclude other causes of her symptoms.
A chest x-ray (choice B) is used to evaluate a cardiopulmonary process. Since
this patient does not have symptoms that seem directly related to these
systems, a chest x-ray is not the most appropriate next step. It may be indicated
in the future because fatigue may be associated with cardiopulmonary
processes, but a biochemical profile should be ordered next.
Fluoxetine (choice C) is used to treat depression and while it is possible that
this patient is depressed, she requires an evaluation with laboratory studies for
her fatigue. She does not complain of any other symptoms of depression, such
as difficulty sleeping, change in concentration, guilt, worthlessness, and
suicidal ideation.
Reassurance and a scheduled follow-up visit in 1 week (choice D) is
inappropriate. An evaluation for her fatigue is indicated because a 3-month
history of "tiredness" is not normal.
Referring her to a psychiatrist (choice E) may be indicated in the future, but at
this time she requires an evaluation for fatigue.
A 47-year-old woman comes to the office with newly diagnosed type 2
diabetes mellitus for a follow-up visit regarding laboratory studies that you had
ordered. She has no complaints at this time. Her temperature 37 C (98.6 F),
blood pressure is 122/80 mm Hg, pulse is 82/min, and weight is 116 kg (255
lb). Visual acuity is 20/20 in both eyes. Her hemoglobin A1C is 6.0% and her
fasting blood glucose is 132 mg/dL. During your discussion with her, she
relates that her father had diabetes and "went blind." She asks you if she should
go to an ophthalmologist. The most appropriate response is:
A. "Good visual acuity is an accurate predictor of the absence of diabetic
retinopathy."
B. "I will perform yearly funduscopic exams and will refer you for
ophthalmology consultation at the first sign of diabetic retinopathy."
C. "Ophthalmology consultation is recommended 5 years after the
diagnosis of diabetes and yearly thereafter."
D. "With well-controlled diabetes you will prevent diabetic visual
complications making a visit to an ophthalmologist unnecessary at this point."
E. "Yearly ophthalmology appointments should begin now."
Explanation:
The correct answer is E. Yearly eye exams by an ophthalmologist are
recommended for all patients with type 2 diabetes mellitus beginning at the
time of diagnosis.
Good visual acuity is not a predictor of the presence or absence of diabetic
retinopathy (choice A).
Funduscopic exams (choice B) should be a part of a primary care physician's
evaluation of a patient with diabetes mellitus; however, the standard of care for
diagnosis and treatment of diabetic retinopathy is under the care of an
ophthalmologist.
Young patients who are newly diagnosed with type 1 diabetes may delay
ophthalmology consultation for 5 years in the absence of visual complaints, but
this does not apply to type 2 patients (choice C).
Good control of diabetes is important in preventing diabetic retinopathy;
however, some patients will have diabetic retinopathy despite excellent control
(choice D). Furthermore, diabetic patients are more likely to have other ocular
diagnoses such as cataracts and glaucoma that are more easily diagnosed and
managed by an ophthalmologist.
An 83-year-old woman with a history of hypertension and osteoarthritis comes
to the office because of abdominal pain with occasional nausea, constipation,
muscle weakness, and fatigue over the last 4 months. She denies taking any
over-the-counter medications or vitamins. She denies weight changes, change
in stool diameter, melena, bright red blood per rectum, or shortness of breath.
You notice in her chart that she had a normal colonoscopy 7 months ago. Her
temperature is 37.0 C (98.6 F), blood pressure is 120/70 mm Hg, pulse is
73/min, and respirations are 13/min. Physical examination is normal except for
mild kyphosis. Her rectal examination is heme negative. Laboratory studies
show:
The most appropriate next step is to
A. determine parathyroid hormone levels
B. determine serum vitamin D levels
C. determine thyroid stimulating hormone levels
D. order a CT scan of the neck
E. order a serum protein electrophoresis
Explanation:
The correct answer is A. This patient has symptomatic hypercalcemia. The
most likely etiology in this patient is hyperparathyroidism. This diagnosis is
made by checking PTH levels. Since her calcium is elevated, a normal or high
PTH level is inappropriate and therefore would help confirm the diagnosis.
Vitamin D (choice B) intoxication is typically secondary to patients taking
large amounts of vitamin D (often for treatment of hypoparathyroidism). The
treatment is to discontinue vitamin supplementation and recheck levels. This
patient denied taking over-the-counter medications or vitamins.
Ordering a TSH (choice C) is never a bad idea in a patient with a variety of
vague complaints since thyroid disease is very common and because it can
explain these symptoms. In this patient, you would expect to find
hypothyroidism based on her symptoms but an elevated serum calcium is more
likely explained by hyperthyroidism secondary to increased bone turnover.
Therefore, a PTH level would more likely lead to the correct diagnosis.
CT scanning of the neck (choice D) is very sensitive for parathyroid disease
but it is not meant to be used as a screening test for hyperparathyroidism.
Rather, the diagnosis is made by checking serum calcium and PTH levels.
Multiple myeloma is a common disease and can cause hypercalcemia as well.
A serum protein electrophoresis (choice E) is helpful in making this diagnosis
by demonstrating a monoclonal spike in the beta or gamma globulin region.
Our patient lacks anemia, renal disease, or significant back pain which makes
multiple myeloma less likely.
A previously healthy 21-year-old woman comes to the local college clinic
because of a headache and low-grade fevers. She is sent home with
acetaminophen and advised to return if she does not improve. Approximately
three hours later her roommate calls 911 reporting that her friend is
unconscious and not arousable. On arrival the paramedics find a lethargic,
febrile female lying on the floor and unresponsive. The patient is stabilized and
she is rushed emergently to the local hospital where an abdominal CT scan
shows bilateral adrenal hemorrhages. The patient is transported to the intensive
care unit where a pulmonary artery catheter is inserted via a right internal
jugular vein. Her temperature is 39.3 C (102.8 F), blood pressure is 85/40 mm
Hg, and pulse is 140/min. An electrocardiogram demonstrates sinus
tachycardia. Her extremities are warm to the touch and appear pink. A cardiac
output (CO), systemic vascular resistance (SVR), pulmonary capillary wedge
pressure (PCWP) and central venous pressure (CVP) are obtained. The data
acquired from her PA-line is most likely to be
A. CO 5.0 L/min, SVR 800 dynes-sec/cm5, PCWP 12 mm Hg, CVP 4 mm
Hg
B. CO 9.5 L/min, SVR 210 dynes-sec/cm5, PCWP 8 mm Hg, CVP 6 mm
Hg
C. CO 2.3 L/min, SVR 1500 dynes-sec/cm5, PCWP 30 mm Hg, CVP 10
mm Hg
D. CO 3.0 L/min, SVR 1500 dynes-sec/cm5, PCWP 8 mm Hg, CVP 1 mm
Hg
E. CO 9.5 L/min, SVR 1210 dynes-sec/cm5, PCWP 12 mm Hg, CVP 6 mm
Hg
Explanation:
The correct answer is B. The patient likely has septic shock that is
characterized by high cardiac output, low systemic resistance, and a relatively
normal contractile function (PCWP) and filling pressures (CVP).
CO 5.0 L/min, SVR 800 dynes-sec/cm5, PCWP 12 mm Hg, CVP 4 mm Hg
(choice A) is a profile of a normal, healthy person.
CO 2.3 L/min, SVR 1500 dynes-sec/cm5, PCWP 30 mm Hg, CVP 10 mm Hg
(choice C) is a profile of cardiogenic shock. This patient has a depressed
cardiac output, a markedly elevated PCWP reflecting failure, and an
appropriately high peripheral vasoconstrictive response to maintain blood
pressure.
CO 3.0 L/min, SVR 1500 dynes-sec/cm5, PCWP 8 mm Hg, CVP 1 mm Hg
(choice D) reflects hypovolemic shock with a low output, an appropriately
high peripheral resistance but very low filling pressures.
CO 9.5 L/min, SVR 1210 dynes-sec/cm5, PCWP 12 mm Hg, CVP 6 mm Hg
(choice E) is representative of a healthy person undergoing vigorous exercise,
with augmented cardiac output, normal systemic resistance and filling
pressures.
A 48-year-old man comes to the clinic with symptoms of sexual dysfunction.
He states that for the last year and a half, he has had a markedly decreased
libido and trouble maintaining an erection. He has also occasionally noticed
some milky-type of discharge from his nipples. He denies headaches, shortness
of breath, or chest pain. He has had no abdominal or urinary symptoms. He has
no significant past medical history and takes no medications. On physical
examination, he is afebrile and has normal vital signs. His visual acuity, visual
fields, extraocular movements, and pupillary response to light are normal.
Remainder of neurologic examination is normal. Laboratory studies show a
leukocyte count of 5,600/mm3, hematocrit 45%, platelets 230,000/mm3,
glucose 100 mg/dL, creatinine 0.8 mg/dl, blood urea nitrogen 16 mg/dl, serum
prolactin 1,000 ng/ml (normal <20 ng/ml). The next most appropriate step in
management is
A. bromocriptine
B. mammography
C. a MRI of the brain
D. a MRI of the lumbar spine
E. sildenafil citrate
Explanation:
The correct answer is C. This patient has impotence, loss of libido, and
galactorrhea related to hyperprolactinemia. The most common cause of this in
men is probably medication induced, however, he is on no medications. The
next main thing to rule out is a prolactin secreting microadenoma by an MRI of
the brain.
Bromocriptine (choice A) may end up being the treatment of choice for this
patient if a microadenoma is found. However the diagnosis should be made
prior to treatment. If bromocriptine therapy fails to relieve symptoms, or if the
adenoma grows/begins causing other symptoms, referral to neurosurgery to
evaluate for transphenoidal resection would be warranted.
There is no indication for a mammogram (choice B) or an MRI of the lumbar
spine (choice D) in this case.
Sildenafil (choice E) also may end up being helpful in this case for the patient's
symptoms. However, again making the underlying diagnosis is the most
appropriate first step in the work up.
A 63-year-old man comes to the office because of "problems seeing." He says
that his wife is making him "get some help" because he got into a minor car
accident last night, and it was his fault because he had difficulty seeing. His
vision is "fine" during the day, but he is basically "blind as a bat" when it is
dark. He admits to an "occasional bottle of vodka." He has dry skin with
multiple, diffuse areas of hyperkeratosis. His condition would most likely have
been prevented by supplementation with
A. niacin
B. vitamin A
C. vitamin B1
D. vitamin B12
E. vitamin C
Explanation:
The correct answer is B. This patient most likely has a vitamin A (retinoic
acid) deficiency, which is characterized by night blindness and dry,
hyperkeratotic skin. It can progress to conjunctival dryness, corneal ulceration,
and necrosis. It occurs in patients with malabsorption or proteinuria, liver
disease, alcoholics, and those receiving total parenteral nutrition (TPN).
Niacin (choice A) deficiency, which is often called pellagra, is characterized
by diarrhea, dementia, and dermatitis. It occurs in individuals with a high
intake of maize (corn). It is not associated with night blindness.
Vitamin B1 (thiamine) deficiency (choice C), which is often called beri-beri, is
characterized by high output heart failure and central nervous system
disturbances (Wernicke-Korsakoff syndrome). Wernicke-Korsakoff syndrome
occurs in alcoholics and is associated with nystagmus, ataxia, confabulation,
and retrograde amnesia. It is not associated with night blindness.
A deficiency of vitamin B12 (choice D) is associated with megaloblastic anemia
and peripheral neuropathy. It is not associated with night blindness. It occurs in
individuals with pernicious anemia, postgastrectomy, and those with intestinal
organisms or ileal abnormalities.
A deficiency of vitamin C (choice E), which is often called scurvy, is
characterized by easy bruising, perifollicular hemorrhages, purpura, poor
wound healing, bone lesions, and emotional changes. It is not commonly
associated with night blindness. It occurs in individuals with diets devoid of
citrus fruits and vegetables.
A 33-year-old man comes to the office complaining of 3 months of severe
headaches preceded by sweating and palpitations. He denies any chest pain or
shortness of breath with these episodes. He has not taken any new medications
and denies excessive caffeine intake and illicit drug use. His temperature is
37.0 C (98.6 F), blood pressure is 120/70 mm Hg, pulse is 78/min, and
respirations are 14/min. Physical and neurological examinations are
unremarkable. Thyroid function tests, a complete blood count, and a 24-hour
urine cortisol are all within normal limits. Urine catecholamines are elevated.
You order a CT scan of the abdomen, which shows an adrenal mass. The most
appropriate next step is to
A. begin therapy with phenoxybenzamine
B. begin therapy with propanolol
C. consult a surgeon for emergent laparoscopic adrenalectomy
D. order serial CT scans to evaluate progression
E. perform a fine needle aspiration of the mass
Explanation:
The correct answer is A. This patient most likely has a pheochromocytoma.
The classic triad of headache, palpitations, and diaphoresis should suggest this
diagnosis. Patients are frequently hypertensive but sometimes the hypertension
is paroxysmal and a normal blood pressure is found on exam. Prior to surgery
this patient must be started on an alpha-blocker (phenoxybenzamine) to avoid
intraoperative hypertensive crisis. Laparoscopic adrenalectomy is the treatment
of choice for a pheochromocytoma.
Treatment with a beta blocker (choice B) is initiated only after a patient is
started on an alpha blocker. Unopposed alpha stimulation can worsen
hypertension in patients with a pheochromocytoma.
Laparoscopic adrenalectomy (choice C) is the treatment of choice for
pheochromocytoma but medical stabilization is essential prior to surgery.
Pheochromocytoma is not an indication for emergent surgery.
Serial CT scans (choice D) are important in the management of nonfunctioning adrenal masses (incidentaloma). When an adrenal mass is
discovered incidentally and it is < 6cm repeat imaging is indicated to ensure
that the mass is not quickly growing. A rapidly growing adrenal mass is
suggestive of malignancy.
Fine needle aspiration (choice E) is an important tool in the workup of an
adrenal mass. However, prior to aspirating any adrenal mass,
pheochromocytoma must be ruled out to avoid precipitation of hypertensive
crisis. Fine needle aspiration is not indicated in the management of
pheochromocytoma since it can exacerbate a hypertensive crisis.
A 29-year-old woman comes to the office because of a 12-pound weight gain
in the past 3 months. She does not seem to think that it is related to a change in
appetite. She has no chronic medical conditions and takes no medications. She
exercises regularly and drinks a glass of wine with dinner each night. She says
that she usually only eats foods that are labeled "low-fat" or "fat free." She has
recently moved to your town to live with her fiancee and just started working
as a kindergarten teacher. She enjoys her job and is very happy in her new
home and with her fiancee. She is 168 cm (5 ft 6 in) tall and weighs 67 kg (148
lb). Her blood pressure is 135/90 mm Hg, pulse is 70/min, and respirations are
14/min. Physical examination is unremarkable. She is picking her cuticles and
tapping her feet during the history and examination. The most appropriate next
step is to
A. ask her if she recently quit smoking
B. determine thyroid stimulating hormone levels
C. order a biochemical profile
D. order a dexamethasone suppression test
E. refer her to a nutritionist
Explanation:
The correct answer is A. In the evaluation of weight gain, the first step is to
perform a thorough history and physical examination. If the patient tells you
that they have not really had a change in appetite and have had a recent weight
gain, you need to ask about medications, such as tricyclics, steroids, and
antipsychotics and about smoking cessation and psychiatric disorders. This
patient appears fidgety, which may be consistent with smoking cessation.
Smoking cessation is often associated with a small weight gain (5-10 lbs), but
it can lead to more dramatic weight gain in others. Before you perform any
expensive and unnecessary diagnostic studies, you should first perform a
detailed history.
Determining thyroid stimulating hormone levels (choice B) is appropriate in
the evaluation of weight gain if there is a negative drug history, no recent
smoking cessation, and a normal biochemical profile. Since hypothyroidism is
part of the differential diagnosis for weight gain, you should consider it early
in the work-up, but a biochemical profile should be performed first.
The initial work-up for weight gain is a detailed history, including medication
and drug use and recent smoking cessation. If those are negative, then a
biochemical profile (choice C) is indicated. A biochemical profile may indicate
the presence of diabetes mellitus or Cushing syndrome. Diabetes will most
likely lead to an elevated glucose, while Cushings is sometimes associated
with hypokalemia, hypochloremia, metabolic alkalosis, hyperglycemia, and
hypercholesterolemia.
A dexamethasone suppression test (choice D) is part of the initial evaluation of
Cushing syndrome. You should first perform a complete history and if
negative for such things as drug history, recent smoking cessation, and a
psychiatric disorder, you should then order a biochemical profile. A
dexamethasone suppression test should be ordered if the biochemical profile is
suspicious for Cushing syndrome (hypokalemia, hypochloremia, metabolic
alkalosis, hypercholesterolemia, and hyperglycemia).
Referring her to a nutritionist (choice E) is inappropriate at this time. The case
says that the patient has had weight gain with no change in appetite or eating
habits. She requires a thorough history and examination, and if negative, an
evaluation for a medical condition such as Cushing syndrome, diabetes
mellitus, thyroid disease (even though this is usually associated with a poor
appetite), and fluid overload. A nutritionist may be valuable later if the
complete evaluation is negative and you determine that she has poor eating
habits. Obviously in that case you should try to help her with her dietary
habits, but if you are unsuccessful, a nutritionist may be helpful.
A 57-year-old woman comes to your office because she is frustrated about the
control of her diabetes. She insists that she has been compliant with her diet
but her sugars continue to be poorly controlled. Her current insulin regimen is
as follows:
14 units NPH and 6 units regular insulin 30 minutes prior to breakfast, 8 units
regular insulin 30 minutes prior to dinner, and 10 units NPH insulin before
bedtime
She has brought along a log of her sugars.
DAY
Sunday
Monday
Tuesday
Wednesday
Thursday
Friday
Saturday
8a.m.**
230*
188
195
197
210
150
306
11a.m.
133
120
100
179
122
120
111
4p.m.
99
103
129
113
147
133
89
9p.m.
300
198
145
139
306
167
203
*Numbers reflect glucose in mg/dl
**8 a.m. sugars are fasting. The most appropriate changes to this patient's
insulin regimen would include
A. decreasing pre-breakfast regular insulin and increasing pre-breakfast
NPH insulin
B. increasing bedtime NPH insulin and increasing dose of pre-dinner
regular insulin
C. increasing NPH insulin and regular insulin doses prior to breakfast
D. increasing pre-breakfast regular insulin and increasing pre-dinner regular
insulin
E. increasing pre-dinner regular insulin dose only
Explanation:
The correct answer is B. This patient has blood sugars which are poorly
controlled in the morning and at 9 p.m. Fasting sugars are a reflection of night
time NPH doses while late morning sugars are a reflection of a.m. regular
doses. Evening sugars reflect dinner time regular insulin. The approximate
onset of action, peak effect, and duration of action of NPH insulin and regular
insulin are shown below:
Type Insulin Duration of action
Onset of Action (hr)
Peak Effect
Peak Effect
Regular
0.5-1
2-4
4-6
NPH
2-4
8-14
16-24
Decreasing pre-breakfast regular insulin and increasing pre-breakfast NPH
insulin (choice A) would increase 11 a.m. sugars and decrease 4 p.m. sugars.
Increasing NPH insulin and regular insulin doses prior to breakfast (choice C)
would be reflected in 11 a.m. and 4 p.m. sugars respectively.
Increasing pre-breakfast regular insulin and increasing pre-dinner regular
insulin (choice D) would be reflected in 11 a.m. and 9 p.m. sugars respectively.
Increasing pre-dinner regular insulin dose only (choice E) would reduce 9 p.m.
sugars only.
This question might seem confusing, but when confronted with a question that
requires that you adjust a patient's insulin regimen it is helpful to answer the
question without looking at any answer choices. You could then simply match
your answer with the answer choices. Again, remember that a.m. regular
insulin will control glucose levels after breakfast. NPH insulin given in the
morning will peak around lunch time and will be reflected in post-lunch
sugars. Pre-dinner regular insulin will obviously help you with night time or
post-dinner sugars and night time NPH will help you control overnight sugars.
If you remember this, this question becomes much less overwhelming.
A 57-year-old woman comes to your office because she is frustrated about the
control of her diabetes. She insists that she has been compliant with her diet
but her sugars continue to be poorly controlled. Her current insulin regimen is
as follows:
14 units NPH and 6 units regular insulin 30 minutes prior to breakfast, 8 units
regular insulin 30 minutes prior to dinner, and 10 units NPH insulin before
bedtime
She has brought along a log of her sugars.
DAY
Sunday
Monday
Tuesday
Wednesday
Thursday
Friday
Saturday
8a.m.**
230*
188
195
197
210
150
306
11a.m.
133
120
100
179
122
120
111
4p.m.
99
103
129
113
147
133
89
9p.m.
300
198
145
139
306
167
203
*Numbers reflect glucose in mg/dl
**8 a.m. sugars are fasting. The most appropriate changes to this patient's
insulin regimen would include
A. decreasing pre-breakfast regular insulin and increasing pre-breakfast
NPH insulin
B. increasing bedtime NPH insulin and increasing dose of pre-dinner
regular insulin
C. increasing NPH insulin and regular insulin doses prior to breakfast
D. increasing pre-breakfast regular insulin and increasing pre-dinner regular
insulin
E. increasing pre-dinner regular insulin dose only
Explanation:
The correct answer is B. This patient has blood sugars which are poorly
controlled in the morning and at 9 p.m. Fasting sugars are a reflection of night
time NPH doses while late morning sugars are a reflection of a.m. regular
doses. Evening sugars reflect dinner time regular insulin. The approximate
onset of action, peak effect, and duration of action of NPH insulin and regular
insulin are shown below:
Type Insulin Duration of action
Onset of Action (hr)
Peak Effect
Peak Effect
Regular
0.5-1
2-4
4-6
NPH
2-4
8-14
16-24
Decreasing pre-breakfast regular insulin and increasing pre-breakfast NPH
insulin (choice A) would increase 11 a.m. sugars and decrease 4 p.m. sugars.
Increasing NPH insulin and regular insulin doses prior to breakfast (choice C)
would be reflected in 11 a.m. and 4 p.m. sugars respectively.
Increasing pre-breakfast regular insulin and increasing pre-dinner regular
insulin (choice D) would be reflected in 11 a.m. and 9 p.m. sugars respectively.
Increasing pre-dinner regular insulin dose only (choice E) would reduce 9 p.m.
sugars only.
This question might seem confusing, but when confronted with a question that
requires that you adjust a patient's insulin regimen it is helpful to answer the
question without looking at any answer choices. You could then simply match
your answer with the answer choices. Again, remember that a.m. regular
insulin will control glucose levels after breakfast. NPH insulin given in the
morning will peak around lunch time and will be reflected in post-lunch
sugars. Pre-dinner regular insulin will obviously help you with night time or
post-dinner sugars and night time NPH will help you control overnight sugars.
If you remember this, this question becomes much less overwhelming.
A 29-year-old man is admitted to the hospital for severe diarrhea. He presented
to the hospital after 3 days of nearly 20 “very watery” bowel movements daily.
He had just recently returned from a trip to the flooded areas of the Carolina
coast. On examination, he appears listless but responsive. His vital signs are
stable. His skin turgor is increased and he has prolonged capillary refill time.
His heart and lung examinations are unremarkable. His serum sodium is 118
mmol/L and urine sodium is less than 10 mmol/L. His serum potassium is 2.9
mmol/L and bicarbonate is 12 mmol/L. The most appropriate therapy is
A. fluid restriction
B. intravenous bicarbonate
C. intravenous hypertonic saline
D. intravenous hypotonic saline
E. intravenous isotonic saline
Explanation:
The correct answer is E. This patient has true hyponatremia and his exam
suggests that his volume status is hypovolemic. A low urine sodium suggests
that he is retaining all of his filtered sodium in an attempt to relieve his
hyponatremia. The therapy is repletion of volume and salt. One half of the total
sodium deficit should be repleted in the first 12 hours, with the remainder
given over the remaining 24 hours. The sodium should rise by no more than
0.5 mEq/L/hour (12 mEq/day). The rate of correction is done in this manner to
decrease the possibility of central pontine demyelinosis.
Fluid restriction (choice A) is the treatment of choice for hypervolemic
hyponatremia (congestive failure or cirrhosis) and with the syndrome of
inappropriate anti-diuretic hormone (SIADH).
Intravenous bicarbonate (choice B) is not necessary to correct this patient's
acid-base deficit. Although his bicarbonate is low, this is likely a result of his
diarrhea and until his pH is known, there is no reason for concern.
There is, in genera, little indication for intravenous hypertonic saline (choice
C) as it is dangerous and can cause great harm when used to correct
hyponatremia if extreme care is not taken.
Intravenous hypotonic saline (choice D) will aggravate the hyponatremia by
giving free water.
You are caring for a patient in the intensive care unit who was admitted 2
hours earlier with diabetic ketoacidosis. She is a 19-year-old girl with no
known medical problems prior to this admission when her parents brought her
in with abdominal pain, nausea, vomiting, and mild confusion. On admission
her laboratory results were as follows:
Sodium
129 mEq/dL
Potassium
5.5 mEq/dL
Chloride
88 mEq/dL
Bicarbonate
12 mEq/dL
Urea nitrogen, serum
51 mg/dL
Creatinine
2.0 mg/dL
Glucose
697 mg/dL
An insulin drip was started at 11 U/hr and intravenous fluids were started at
250 cc/hr. Two hours later, laboratory studies show:
Sodium
132 mEq/dL
Potassium
4.0 mEq/dL
Chloride
89 mEq/dL
Bicarbonate
13 mEq/dL
Urea nitrogen, serum
41 mg/dL
Creatinine
1.9 mg/dL
Glucose
277 mg/dL
Shortly after these laboratory results return, the patient becomes unresponsive.
Papilledema is observed bilaterally. The most likely explanation for the
patient's rapid deterioration is
A. arterial thrombosis associated with diabetic ketoacidosis
B. overlooking of toxic screen in patients initial evaluation
C. rapid lowering of serum glucose
D. use of a relatively hypertonic solution for rehydration
E. worsening lactic acidosis and tissue ischemia
Explanation:
The correct answer is C. The target for lowering serum glucose is
approximately 50-100 mg/dL/hour. More aggressive lowering of the serum
glucose can result in cerebral edema. This is because the CSF glucose
decreases more slowly than the serum glucose. If the serum glucose is lowered
too rapidly, the CSF fluid is relatively hypertonic compared to the serum.
Therefore, water will enter the CSF creating edema.
Arterial thrombosis (choice A) occurs with increased frequency in DKA. This
can manifest as MI, ischemic limbs, or stoke. The fact that this patient has
papilledema suggests that edema is more likely than stroke. This patient's age
also makes stroke less likely.
Toxic screening for drugs should be part of the initial evaluation of a patient
with altered mental status. This patient has an anion gap acidosis with an
elevated glucose and likely does have DKA. Therefore, a toxic drug screen
(choice B) is not the best answer.
Normal saline (choice D) is an appropriate solution for rehydration in this
patient. Although the patient's sodium is 129 mEq/dl on admission, when
corrected for the elevated glucose, the sodium is 139 mEq/dL. Remember that
these patients are very dehydrated on admission and need to be volume
resuscitated.
Lactic acidosis (choice E) can occur in patients with DKA from prolonged
dehydration, infection, or tissue hypoxia. One would suspect that if this patient
had a lactic acidosis on admission that it would correct with proper DKA
management. You would not expect brain herniation from lactic acidosis.
A 44-year-old woman comes to see you for routine check up. While in your
office, she starts crying hysterically, stating she has not been able to sleep for
the last few months. Also, her mind has been "racing," her palms and soles are
sweaty at all times, and she has thinning of her hair. She denies any alcohol or
tobacco use, but admits to 2 cups of coffee a day. Her mother has bipolar
disorder and her aunt has obsessive-compulsive personality disorder. Her
blood pressure is 130/80 mm Hg and pulse is 100/min. There is notable
exophthalmos bilaterally. He skin appears moist and warm. On the pretibial
regions, there is a woody induration with pitting edema. The most appropriate
next step in evaluation is to
A. determine testosterone level
B. give her aluminum chloride for her sweaty palms and reassure her this is
all psychological
C. order thyroid function tests
D. refer her to an ophthalmologist for evaluation of the exophthalmos
E. refer her to a psychiatrist for evaluation of a psychiatric disorder
Explanation:
The correct answer is C. Ordering a thyroid function test is correct, because
this patient demonstrates not only physiological hyperthyroid changes (i.e.
increased heart rate), but also cutaneous findings classic for Graves disease.
Skin changes are distinctive in hyperthyroidism. The cutaneous surface is
warm, moist, and smooth textured. Palmar erythema or facial flushing may be
seen. The hair is thin and has a downy texture and nonscarring alopecia may be
observed. Graves disease has a female to male ratio of 7:1. Thyroid acropachy
is characterized by digital clubbing and diaphyseal proliferation of the
periosteum in acral and distal long bones (tibia, fibula, ulna, and radius).
Pretibial myxedema consists of bilateral localized, cutaneous accumulations of
glycosaminoglycans and occurs in 4% of patients who have or have had
Graves disease. Improvement in plaques of pretibial myxedema have resulted
from intralesional injections of triamcinolone acetonide and with clobetasol
solution under Duoderm occlusion, applied once weekly for 4-6 weeks.
Systemic steroids are of no benefit.
Checking testosterone level (choice A) is incorrect, because an elevated
testosterone level would only account for hair thinning, but not her other
physiologic and cutaneous changes.
Giving her aluminum chloride (choice B) is incorrect, because this solution
will only alleviate sweaty palms and soles for this patient, but does not help to
diagnose and treat the underlying issue.
Referring her to ophthalmology (choice D) is incorrect, because eventually this
patient may need to see an ophthalmologist for severe exophthalmos, but
diagnosing Graves disease is the more appropriate first step for this patient.
Despite her family history of psychiatric illnesses, referral to psychiatry
(choice E) is incorrect, because this patient's manic appearance is most likely
secondary to thyroid hormone imbalance.
A 70-year-old man with hypertension, hyperlipidemia, and chronic atrial
fibrillation is brought to the emergency department for confusion. He was
recently diagnosed with multiple myeloma. His medications include
furosemide, captopril, atorvostatin, digoxin, and warfarin. He is allergic to
penicillin to which he gets a rash. His temperature is 37.0 C (98.6 F), blood
pressure is 100/60 mmHg, pulse is 98/min, and respirations are 23/min.
Physical examination shows an irregular cardiac rhythm and a soft systolic
murmur at his cardiac base. An electrocardiogram shows atrial fibrillation.
Laboratory studies show:
Sodium
143 mEq/L
Potassium
4.5 mEq/L
Chloride
104 mEq/L
Bicarbonate
26 mEq/L
Calcium
13 mg/dL
Glucose
109 mg/dL
The most appropriate next step is management is to
A. administer albumin, intravenously
B. administer a dextrose bolus followed by insulin, intravenously
C. administer magnesium sulfate, intravenously
D. administer pamidronate, intravenously
E. hydrate him with normal saline and then administer furosemide,
intravenously
Explanation:
The correct answer is E. The most appropriate acute treatment of
hypercalcemia is hydration followed by a forced diuresis. It is important to
hydrate patients prior to administering the diuretic since most patients with this
condition are hypovolemic from hypercalcemia induced nausea/vomiting and
diabetes insipitus.
Administration of albumin (choice A), albeit a binder of serum calcium, has no
role in the management of hypercalcemia.
Administration of dextrose followed by insulin (choice B) is one of the
treatments of choice for acute hyperkalemia. It has no role in the management
of hypercalcemia.
Administration of magnesium sulfate (choice C) has no role in the
management of hypercalcemia.
Administration of pamidronate (choice D), a bisphosphonate which can
decrease bone resorption, can be used in the chronic management of
hypercalcemia, but has no role in its acute management.
A 23-year-old woman comes to the clinic for a pre-employment examination.
She recently moved to the area from out of state and got a job at a local small
business. Her past medical history is significant only for diabetes mellitus type
I, which she has had since age 13. Her only medication is insulin, which is
infused via an insulin pump. She denies smoking or using illicit drugs. She
admits to social alcohol consumption less than once a week and says she runs 2
miles daily. She is not sexually active. You perform a full physical
examination. Her temperature is 37.1 (98.8 F), blood pressure is 136/89 mm
Hg, pulse is 54/min, and respirations are 12/min. Her skin is warm and dry.
Cardiovascular examination reveals a normal S1, S2 with no murmurs
appreciated. Respirations are equal bilaterally without any abnormal breath
sounds. Extremities show no clubbing, cyanosis, or edema. Strength is equal
bilaterally and sensation is full throughout. The patient exhibits normal
reflexes. She returns to the clinic several more times and her blood pressure
remains elevated. The most appropriate initial pharmacotherapy for this patient
is
A. amlodipine
B. atenolol
C. enalapril
D. furosemide
E. hydrochlorothiazide
Explanation:
The correct answer is C. This patient is a type I diabetic with mild
hypertension. The Joint National Committee recommends ACE inhibitors,
such as enalapril, as first-line treatment of hypertension in diabetics. Multiple
studies have proven that ACE inhibitors prevent microalbuminuria and thus
preserve renal function.
Amlodipine (choice A) is a calcium channel blocker commonly prescribed for
hypertension. However, it has not been proven to have the renal protective
properties of ACE inhibitors. Therefore, choosing amlodipine is incorrect.
Atenolol (choice B) is a beta-blocker that is often used as a first-line agent in
non-diabetics. However, beta-blockers should be avoided in patients with type
I diabetes, asthma, and depression. Additionally, the patient's low heart rate
would contraindicate the use of a beta-blocker.
Furosemide (choice D) and hydrochlorothiazide (choice E), a thiazide diuretic
and a loop diuretic, are often used as initial agents in the treatment of
hypertension due to their efficacy and cost effectiveness. However, the renal
protective benefits of an ACE inhibitor make enalapril a more appropriate
choice for this patient.
A 20-year-old college student is brought to the emergency department by his
girlfriend because of the sudden onset of a headache, shaking, sweating, and
blurry vision. She says that he became very confused during the taxicab ride
over to the hospital, asking, "where are you taking me to, the airport?" He was
fully aware that they were going to the hospital as they left their apartment 5
minutes earlier. These symptoms started as they were lying in bed going to
sleep, 3 hours after coming back from the local Italian restaurant, where they
both ate fettucine alfredo. They did not drink any alcohol tonight. He has had
similar, but milder, symptoms on four previous occasions in the past few years.
He has no other medical conditions and does not take any medications. His
temperature is 37.0 C (98.6 F), blood pressure is 100/70 mm Hg, pulse is
120/min, and respirations are 22/min. He is not oriented to person, place, or
time. He begins to lose consciousness during the physical examination, which
is otherwise unremarkable. At this time the most appropriate conclusion is:
A. He should drink a glass of orange juice immediately
B. An intravenous bolus of glucose as a 50 % solution should be given
immediately
C. An intravenous bolus of isotonic saline should be given immediately
D. Naloxone, given intravenously every 2 minutes, will reverse this
patient's symptoms
E. Physostigmine, given intravenously over 2 minutes, will reverse this
patient's symptoms
Explanation:
The correct answer is B. This patient is most likely experiencing the symptoms
of hypoglycemia, which should be treated with intravenous glucose until the
patient can eat a meal. There are two categories of symptoms associated with
hypoglycemia: the autonomic response and the neuroglycopenic response. The
former is due to excessive secretion of epinephrine/norepinephrine and consists
of tremor, palpitations, sweating, hunger, and anxiety. The latter is caused by
central nervous system dysfunction and includes dizziness, headache, blurry
vision, confusion, abnormal behavior, and a loss of consciousness. Symptoms
do not usually occur until the blood glucose falls below 45 mg/dL. Causes of
hypoglycemia include postprandial hypoglycemia, fasting hypoglycemia,
insulinomas, and factitious hypoglycemia.
This patient should not have a glass of orange juice (choice A) right now
because he is losing consciousness and therefore cannot eat or drink. The
initial treatment for hypoglycemia with both autonomic and neuroglycopenic
symptoms is intravenous glucose. This can be discontinued when the patient is
able to eat and drink.
Administration of isotonic saline, intravenously (choice C) is part of the initial
treatment for diabetic ketoacidosis. 5% glucose solutions should be added
when the plasma glucose level falls below 300 mg/dL. This would be
inappropriate for a hypoglycemic because they need glucose. The symptoms of
DKA include increased urination, thirst, abdominal pain, mental status
changes, anorexia, nausea, and vomiting. The patient in this case does not have
these symptoms.
Naloxone would reverse this patient's symptoms (choice D) if they were
caused by an opioid overdose, not hypoglycemia. Naloxone is an opioid
antagonist that competes for the opioid receptors and will block the effects of
heroin and other opioids. Symptoms of an opioid overdose include miosis,
respiratory depression, coma, hypotension, and bradycardia. The patient in this
case does not have these symptoms.
Physostigmine would reverse this patient's symptoms (choice E) if they were
caused by anticholinergic poisoning, not hypoglycemia. The symptoms of
anticholinergic poisoning can occur with overdoses of antihistamines, atropine,
tricyclic antidepressants, and scopolamine, and include hyperthermia,
vasodilatation, decreased salivation, mydriasis, delirium, and hallucinations.
The girlfriend did not say that he took any medications. The fact that these
symptoms occurred a few hours after a meal and that he has had similar
episodes in the past and his physical examination did not reveal mydriasis or
hyperthermia makes his condition more consistent with hypoglycemia than
anticholinergic poisoning.
A 31-year-old man with insulin dependent diabetes mellitus is admitted to the
hospital because of a severe diabetic crisis. His wife reports that over the past
few days he has developed "the flu." During that time, his blood sugars had
become much more difficult to manage despite diligent attention. She states
that over the past 24 hours his sugars have been above 500 mg/dL despite
insulin and diet control, that have always controlled the sugars in the past. The
patient began to breathe very fast in the last 8 hours and he has become
somewhat confused in the past 4 hours. His temperature is 38.0 C (99.6 F),
blood pressure is 140/85 mm Hg, pulse is 88/min, and respirations are 24/min.
His urine is 4+ for ketones and sugar. A fingerstick blood glucose reveals a
blood sugar of 850 mg/dL. An arterial blood gas would most likely show:
A. PaCO2 14 mm Hg, pH 7.12, PaO2 60 mm Hg
B. PaCO2 14 mm Hg, pH 7.22, PaO2 90 mm Hg
C. PaCO2 14 mm Hg, pH 7.38, PaO22 90 mm Hg
D. PaCO22 35 mm Hg, pH 7.36, PaO2 90 mm Hg
E. PaCO2 68 mm Hg, pH 6.80, PaO2 60 mm Hg
Explanation:
The correct answer is B. When looking at arterial blood gases, examine the pH
to identify the acid-base disturbance and then determine whether the acid-base
disturbance is respiratory (change in CO2) or metabolic. The relationship
between PaCO2 and pH determines whether the condition is acute or chronic.
Chronic conditions have a pH closer to 7.4 than would be predicted based upon
PCO2 because of compensation. This patient has diabetic ketoacidosis. This
condition occurs when some event "pushes" the patient over the edge and they
are unable to regulate glucose balance. Typical events include infection or
stress. For this man, his primary problem is a metabolic acidosis that will cause
him to hyperventilate to compensate for. His oxygenation will be essentially
normal since he has no pulmonary pathology: PaCO2 14 mm Hg, pH 7.22, and
PaO2 90 mm Hg.
A PaCO2 of 14 mm Hg, pH of 7.12, and PaO2 of 60 mm Hg (choice A) reflects
a profound metabolic acidosis with hypoxemia. This is not DKA but perhaps a
condition such as sepsis or cyanide poisoning.
A PaCO2 of 14 mm Hg, pH 7.38, and PaO2 90mm Hg (choice C) represents a
chronic acidosis. The pH is mildly acidic with a very low PaCO2. For this
PaCO2, the patient should be alkalotic, but since he is not, it must have
increased the pH from a very low level to near normal. This is the hallmark of
a compensated metabolic acidosis. This can be seen with conditions such as
Type I and II renal tubular acidosis.
A PaCO2 of 35 mmHg, pH 7.36, and PaO2 of 90 mm Hg (choice D) is a
compensated metabolic acidosis. The PaCO2 is nearly normal which suggests
the acidosis is mild. Such mild acidosis can occur with specific renal tubular
disorders or with chronic diarrhea.
A PaCO2 of 68 mmHg, pH 6.80, and PaO2 of 60 mm Hg (choice E) represents
a combined metabolic/respiratory acidosis. The patient has a severe acidosis
(pH), the expected PaCO2 should be low to compensate. Instead, it is high. The
patient also has hypoxemia. This is typical of severe respiratory failure.
Hypoxia causes a metabolic acidosis and the pulmonary system cannot
compensate.
A 77-year-old man with comes to the emergency department with left knee
swelling and intense pain for 7 hours. He has a history of hypertension, treated
with hydrochlorothiazide, alcohol abuse, and chronic renal insufficiency with a
baseline creatinine of 3.4 mg/dL. His temperature is 37.3 C (99.2 F). Physical
examination shows an erythematous, warm, tender knee. The remainder of the
examination is unremarkable. You aspirate fluid from his knee and send it for
evaluation. The results come back as "negatively birefringent crystals." The
most appropriate management at this time is to administer
A. allopurinol
B. aspirin
C. colchicine
D. indomethacin
E. intraarticular steroids
Explanation:
The correct answer is E. Steroids, especially intraarticular steroids, are very
effective when NSAIDs (which are the treatment of choice for gout) are
contraindicated. In this patient, chronic renal insufficiency is a relative
contraindication to NSAID use. Therefore, intraarticular steroids would likely
give him relief from his pain by decreasing inflammation and provide
treatment with few systemic effects.
Allopurinol (choice A) is a xanthine oxidase inhibitor, which is an effective
therapy for hyperuricemia. Allopurinol has no role in the treatment of acute
gout since it may cause the mobilization of tophi, thereby causing increased
pain. It is useful to keep the uric acid level within normal limits and should be
started after the acute attack has resolved to prevent recurrences. Do not forget
to keep a patient's comorbidity in mind prior to prescribing treatments. This
patient would likely benefit from decreasing his alcohol intake and changing
his antihypertensive medication to one that will not increase uric acid levels.
Aspirin (choice B), especially low-dose aspirin, may increase uric acid levels
by inhibiting renal excretion of uric acid. Aspirin is therefore not indicated in
the treatment of gout.
Colchicine (choice C) is an effective treatment for gout but there are several
side effects associated with its use. It is often associated with severe GI side
effects. At high doses (especially IV therapy), bone marrow suppression can
occur. Liver disease and kidney disease may increase colchicine levels and
should be avoided in these patients if possible.
NSAIDs, such as indomethacin, (choice D) are the treatment of choice for
most patients with gout. Typically, treatment begins with high dose NSAIDs,
which are rapidly tapered. This patient has kidney disease and therefore should
not be given high-dose NSAIDs.
A 44-year-old woman comes to see you for routine check up. While in your
office, she starts crying hysterically, stating she has not been able to sleep for
the last few months. Also, her mind has been "racing," her palms and soles are
sweaty at all times, and she has thinning of her hair. She denies any alcohol or
tobacco use, but admits to 2 cups of coffee a day. Her mother has bipolar
disorder and her aunt has obsessive-compulsive personality disorder. Her
blood pressure is 130/80 mm Hg and pulse is 100/min. There is notable
exophthalmos bilaterally. He skin appears moist and warm. On the pretibial
regions, there is a woody induration with pitting edema. The most appropriate
next step in evaluation is to
A. determine testosterone level
B. give her aluminum chloride for her sweaty palms and reassure her this is
all psychological
C. order thyroid function tests
D. refer her to an ophthalmologist for evaluation of the exophthalmos
E. refer her to a psychiatrist for evaluation of a psychiatric disorder
Explanation:
The correct answer is C. Ordering a thyroid function test is correct, because
this patient demonstrates not only physiological hyperthyroid changes (i.e.
increased heart rate), but also cutaneous findings classic for Graves disease.
Skin changes are distinctive in hyperthyroidism. The cutaneous surface is
warm, moist, and smooth textured. Palmar erythema or facial flushing may be
seen. The hair is thin and has a downy texture and nonscarring alopecia may be
observed. Graves disease has a female to male ratio of 7:1. Thyroid acropachy
is characterized by digital clubbing and diaphyseal proliferation of the
periosteum in acral and distal long bones (tibia, fibula, ulna, and radius).
Pretibial myxedema consists of bilateral localized, cutaneous accumulations of
glycosaminoglycans and occurs in 4% of patients who have or have had
Graves disease. Improvement in plaques of pretibial myxedema have resulted
from intralesional injections of triamcinolone acetonide and with clobetasol
solution under Duoderm occlusion, applied once weekly for 4-6 weeks.
Systemic steroids are of no benefit.
Checking testosterone level (choice A) is incorrect, because an elevated
testosterone level would only account for hair thinning, but not her other
physiologic and cutaneous changes.
Giving her aluminum chloride (choice B) is incorrect, because this solution
will only alleviate sweaty palms and soles for this patient, but does not help to
diagnose and treat the underlying issue.
Referring her to ophthalmology (choice D) is incorrect, because eventually this
patient may need to see an ophthalmologist for severe exophthalmos, but
diagnosing Graves disease is the more appropriate first step for this patient.
Despite her family history of psychiatric illnesses, referral to psychiatry
(choice E) is incorrect, because this patient's manic appearance is most likely
secondary to thyroid hormone imbalance.